Financial Performance Metrics- Financial Ratios

Published on January 2017 | Categories: Documents | Downloads: 354 | Comments: 0 | Views: 3269
of 98
Download PDF   Embed   Report

Comments

Content

Question 1:
2A2-CQ48

Collins Company reported net income of $350,000 for the year. The company had 10,000
shares of $100 par value, non-cumulative, 6% preferred stock and 100,000 shares of $10 par
value common stock outstanding. There were also 5,000 shares of common stock in treasury
during the year. Collins declared and paid all preferred dividends as well as a $1 per share
dividend on common stock. Collins' earnings per share of common stock for the year was:
*Source: Retired ICMA CMA Exam Questions.

$2.90.
$3.50.
$2.76.
$3.33.

Formula = (Net income − Preferred Dividends) / Weighted Average shares outstanding.
(350,000 net income − 60,000 [10,000(100)(.06)] preferred dividend) / 100,000 common
shares = 290,000/100,000 = $2.90 earnings per share of common stock.
Question 2:
2A2-LS34

AEW Corporation 's net income is $5,000,000, and its dividend payout is $1.20 per share, with
500,000 shares outstanding. Its average common shareholders' equity is $7,000,000. What is
AEW's dividend payout ratio?
0.10.
0.12.
0.8.
0.714.

AEW Corporation's net income is $10 per share ($5,000,000 net income ÷ 500,000 shares
outstanding), and its dividend payout is $1.20 per share. Its dividend payout ratio is its
dividend payout per share divided by income per share, or 0.12 ($1.20 dividend per share ÷
$10 income per share).
Question 3:
2A2-CQ29

The following information concerning Arnold Company's common stock was included in the
company's financial reports for the last two years.

Based on the price-earnings information, investors would most likely consider Arnold's
common stock to:
be overvalued at the end of Year 2.
show a positive trend in growth opportunities in Year 2 compared to Year 1.
show a decline in growth opportunities in Year 2 compared to Year 1.
indicate inferior investment decisions by management in Year 2.

The company's P/E (price/earnings) ratio increased from year 1 to year 2. The P/E ratio is
calculated by taking the market price per share and dividing it by the earnings per share.
P/E ratio = (market price per share) / (earnings per share)
P/E ratio, year 1 = $50 / $3 = 16.67
P/E ratio, year 2 = $60 / $3 = 20
Since the P/E ratio is increasing from year 1 to year 2, it is showing a positive trend in growth
opportunities in year 2 compared to year 1.
Question 4:
2A2-LS23

ROK Company has $900,000 in current assets, $200,000 in current liabilities, $1,800,000 in
fixed assets, and $400,000 in long-term liabilities. What is its fixed assets to equity capital
ratio?
0.9.
0.86.
1.28.
0.43.

The fixed assets to equity capital ratio is calculated by dividing fixed assets by shareholders'
equity. Shareholders' equity is assets less liabilities, or $2,100,000 ($2.7 million assets −
$600,000 liabilities). Therefore, the company's fixed assets to equity capital ratio is 0.8571
($1,800,000/$2,100,000).
Question 5:
2A2-AT36

Devlin Company's rate of return on assets (ROA) for the year ended May 31, Year 2, was:
7.8%.
7.2%.
11.2%.
7.5%.

ROA is calculated by taking net income and dividing it by average total assets. Average total
assets is equal to the asset balance at the beginning of the period, plus the asset balance at
the end of the period, all divided by 2 (in order to arrive at an average for the year).

Average total assets = (beginning asset balance + ending asset balance) / 2
Devlin's average total assets = ($748 + $691)/2 = $1439/2 = $719.50
Since net income is $54, then ROA is calculated as:
ROA = net income / average total assets
ROA = $54 / $719.50 = 0.075, or 7.5%
ROA is also called return on investment (ROI).
Question 6:
2A2-CQ19

A financial analyst with Mineral Inc. calculated the company's degree of financial leverage
(DFL) as 1.5. If net income before interest increases by 5%, earnings to shareholders will
increase by:
3.33%.
1.50%.
7.50%.
5.00%.

The DFL is calculated by taking the percent change in earnings to shareholders divided by the
percent change in earnings before interest and taxes (EBIT).
DFL is given as 1.5. DFL measures how an increase in EBIT will affect earnings to
shareholders. Therefore, a 5% increase in EBIT will cause a 7.5% (1.5 × 5%) increase in
earnings to shareholders.
Question 7:
2A2-CQ24

Easton Bank has received loan applications from three companies in the computer service
business and will grant a loan to the company with the best prospect of fulfilling the loan
obligations. Specific data, shown below, has been selected from these applications for review
and comparison with industry averages.

Based on the information above, select the strategy that would fulfill Easton's objective.
Grant the loan to SysGen as the company has the highest net profit margin and degree of financial
leverage.
Easton should not grant any loans as none of these companies represents a good credit risk.
Grant the loan to CompGo as all the company's data approximate the industry average.
Grant the loan to Astor as both the debt/equity (D/E) ratio and degree of financial leverage (DFL) are
below the industry average.

The D/E ratio and the DFL both measure an organization's risk. The lower the D/E ratio, the
lower the risk. Similarly, the lower the DFL, the lower the risk. In the case of Astor, both the
D/E ratio and DFL measures are below the industry averages. Therefore, it is the least risky of
the three choices.
Question 8:
2A2-LS54

When reviewing a credit application, the credit manager should be most concerned with the
applicant's:
*Source: Retired ICMA CMA Exam Questions.

price-earnings ratio and current ratio.
working capital and return on equity.
working capital and current ratio.
profit margin and return on assets.

Liquidity measures, such as net working capital and the current ratio, help determine ability to
pay expenses on a timely basis. Therefore, the credit manager should be most concerned
with these measures in comparison to the others listed in the problem. Profit margin, priceearnings ratio, and return on equity are profitability measures.
Question 9:
2A2-AT28

CPZ Enterprises had the following account information.

The company has an operating cycle of five months.

What will happen to the ratios below if CPZ Enterprises uses cash to pay 50% of the accounts
payable, and both ratios are greater than 1.0 prior to the payment?
current ratio: increase; quick ratio: increase.
current ratio: decrease; quick ratio: increase.
current ratio: increase; quick ratio: decrease.
current ratio: decrease; quick ratio: decrease.

The current ratio is the ratio of current assets to current liabilities. The acid-test ratio (quick
ratio) is the ratio of quick assets to current liabilities.
Current assets consist of cash, short-term marketable securities, accounts receivable, shortterm notes receivable, inventories, and prepaid expenses. Current liabilities are those due
within one year, or one business cycle, whichever is longer. Current liabilities consist of
accounts payable, accrued expenses payable, interest payable, taxes payable, short-term
notes payable, and the current portion of long-term debt.
The quick assets are cash, short-term investments (marketable securities), and receivables.
The cash payment on accounts payable will decrease the current and quick assets by the
same amount. Since both the current ratio and the quick ratio are greater than one, the
change will increase both ratios.
For example, if prior to the payment, the current assets were equal to $200, the quick assets
were $150, and the current liabilities were $100, then the original current ratio would be
$200/$100 = 2 and the original quick ratio would be $150/$100 = 1.5.
Assuming that the current liabilities are all accounts payable, the payment on 50% of the
accounts payable balance would reduce the current assets to $150, the quick assets to $100,
and the current liabilities to $50. The new current ratio would then be $150/$50 = 3. The new
quick ratio would be $100/$50 = 2. By using this example, one can see that both the current
ratio and quick ratio have increased as a result of CPZ Enterprises using cash to pay off 50%
of the account payable balance.
Question 10:
2A2-CQ16

The following information was obtained from a company's financial statements.

Total sales for the year were $85,900, of which $62,400 were credit sales. The cost of goods
sold (COGS) was $24,500.

The company's payable turnover was:
17.8 times.
6.7 times.
7.3 times.
8.6 times.

The accounts payable turnover is calculated as:
Accounts payable turnover = (purchases for the year) / (average accounts payable balance)
Purchases = cost of goods sold + ending inventory − beginning inventory
Purchases = $24,500 + $7,600 − $6,400 = $25,700
Average accounts payable = (beginning balance + ending balance) / 2
Average accounts payable = ($3,320 +$3,680) / 2 = $7,000 / 2 = $3,500
Accounts payable turnover = $25,700 / $3,500 = 7.34, or 7.3 times.
Question 11:
2A2-CQ38

Shown below are selected data from Fortune Company's most recent financial statements.

What is Fortune's net working capital?
*Source: Retired ICMA CMA Exam Questions.

$45,000.
$35,000.
$50,000.
$80,000.

Net working capital is calculated as total current assets minus current liabilities. Therefore
(10,000 + 60,000 + 25,000 + 5,000) − (40,000 + 10,000 + 5,000) = $45,000.
Question 12:
2A2-CQ06

Marge Halifax, chief financial officer of Strickland Construction, has been tracking the
activities of the company's nearest competitor for several years. Among other trends, Halifax
has noticed that this competitor is able to take advantage of new technology and bring new
products to market more quickly than Strickland. In order to determine the reason for this,
Halifax has been reviewing the following data regarding the two companies.

On the basis of this information, which one of the following is the best initial strategy for
Halifax to follow in attempting to improve the flexibility of Strickland?
Seek cost cutting measures that would increase Strickland's profitability.
Increase Strickland's investment in short-term securities to increase the current ratio.
Investigate ways to improve asset efficiency and turnover times to improve liquidity.
Seek additional sources of outside financing for new product introductions.

Seeking additional sources of outside financing increases the debt of the firm, therefore
increasing the numerator in the formula for debt-to-equity, thus increasing the ratio to be more
in-line with competition. Also, increasing the assets through the introduction of new products
(inventory) would increase the Degree of Financial Leverage (Assets / Equity), again, bringing
the financial leverage of Strickland more in line with their competition.
Question 13:
2A2-LS45

Which of the following statements is not correct concerning the price to earnings (PE) ratio?
PEs tend to be larger in industries experiencing high growth.
PEs tend to vary a great deal; however, they tend to be similar within industries.
Assume that a firm gives stock options to its managers and net income and the PE ratio remain
relatively steady. The result of the stock option issuance would be an increase in stock price.
The PE ratio is a measure of investor confidence in the management of a firm. A high ratio implies
greater confidence.

The PE ratio is defined as the market price of the stock divided by fully diluted earnings per
share (EPS). If a firm issues stock options, fully diluted EPS should decrease. If the firm's PE
ratio is fairly constant, then the firm's stock price must fall.
Question 14:
2A2-AT16

When comparing current year financial ratios to prior years, you find that Charley, Inc., has an
improving current ratio and a deteriorating acid test (quick) ratio. Which of the following
actions would be a logical management action step given these results?
Offer cash discounts for early payment on accounts receivable.
Increase budget controls so that expenses are reduced.
Reduce long-term debt by issuing new stock.
Work with suppliers on reducing inventory levels.

The current ratio is current assets divided by current liabilities. The acid-test ratio (also known
as the quick ratio) is the ratio of quick assets to current liabilities.
Current assets consist of cash, short-term marketable securities, accounts receivable, shortterm notes receivable, inventories, and prepaid expenses.
Current liabilities are those due within one year, or one business cycle, whichever is longer.
Current liabilities include accounts payable, accrued expenses payable, interest payable,
taxes payable, short-term notes payable, and the current portion of long-term debt.
The quick assets are cash, short-term investments (marketable securities), and receivables.
An improving current ratio and a deteriorating acid test (quick) ratio would indicate that
Charley, Inc. is increasing its inventory at a rate faster than necessary for the growth of the
business. Charley, Inc. can begin to remedy the problem by working with suppliers to reduce
the inventory levels.
Question 15:
2A2-LS12

Assume that a firm generally has solvency ratios very close to industry averages. An increase
in which of the following ratios would generally be considered a good sign when considering
the solvency of a firm?
Times interest earned.
Financial leverage index.
Debt-to-equity ratio.
Current ratio.

The current ratio is considered a liquidity ratio and not a solvency ratio. An increase in the
financial leverage index or the debt-to-equity ratio implies greater reliance on debt. An
increase in times interest earned implies that the firm is better able to make debt payments
and is therefore considered a good change in financial health.
Question 16:
2A2-AT23

The following financial information applies to Sycamore Company.

What is the acid-test (or quick) ratio for Sycamore?
1.97.
5.63.
2.13.
1.56.

Quick ratio = (cash + marketable securities + accounts receivable)/(accounts payable +
current portion of long-term debt) = ($10,000 + $18,000 + $120,000)/($75,000 + $20,000) =
1.558, which would round to 1.56.
Question 17:
2A2-CQ42

Interstate Motors has decided to make an additional investment in its operating assets which
are financed by debt. Assuming all other factors remain constant, this increase in investment
will have which one of the following effects?

*Source: Retired ICMA CMA Exam Questions.

Increase; No Change; Decrease.
No Change; Decrease; Decrease.
No Change; Increase; Decrease.

Decrease; Decrease; Decrease.

A purchase of assets would have no effect on operating income margin (operating income
divided by sales). It would decrease both operating asset turnover (sales divided by average
total assets) and return on operating assets (operating income divided by average total
assets) by increasing assets.
Question 18:
2A2-LS07

Current statements for LMN Company reveal the following:
1. beginning and ending inventory balance of $730,000 and $770,000, respectively,
2. sales of $15 million, and,
3. a gross profit ratio of 66 2/3%.
What is LMN Company's days' sales in inventory ratio?
9.2 days.
6.67 days.
54.75 days.
15 days.

Before computing the days' sales ratio, one needs to find average inventory (the average of
$730,000 and $770,000, which is $750,000) and then find cost of goods sold (COGS). Since
the gross profit rate is 66 2/3%, gross profit must be $10 million. Since gross profit is sales
less COGS and gross profit is $10 million, then COGS must be $5 million. The calculation for
days' sales in inventory ratio is:

Question 19:
2A2-AT22

A financial analyst has obtained the following data from Kryton Industries' financial
statements.

In order to determine Kryton's ability to pay current obligation, the financial analyst would
calculate Kryton's cash ratio as:

0.50.
1.20.
0.80.
1.00.

Cash ratio = (Cash + Cash Equivalents + Marketable securities) / Current liabilities =
($200,000 + 0 + $100,000) / $600,000 = 0.50.
Question 20:
2A2-LS37

XYB Company has a book value of $25 per share, net income of $3.3 million, annual
preferred dividends of $300,000, and 1 million common stock shares outstanding. The
company's common stock has a current market price of $45. What is the price to book value
ratio?
0.12
0.56
0.067
1.8

The price to book value ratio is calculated by dividing current market price by the book value
per share. $45 current market price ÷ $25 book value per share = 1.8.
Question 21:
2A2-CQ28

The assets of Moreland Corporation are presented below.

For the year just ended, Moreland had net income of $96,000 on $900,000 of sales.
Moreland's total asset turnover ratio is:
1.48.
1.37.
1.50.
1.27.

Total asset turnover is calculated as:
Total asset turnover = (net sales) / (average total assets for the year)
Average total assets = (beginning asset balance + ending asset balance) / 2
Total assets = cash + marketable securities + accounts receivable + inventory + plant &
equipment
Beginning total assets = ($48,000 + $42,000 + $68,000 + $125,000 + $325,000) = $608,000
Ending total assets = ($62,000 + $35,000 + $47,000 + $138,000 + $424,000) = $706,000
Average fixed assets = ($608,000 + $706,000) / 2 = $1,314,000 / 2 = $657,000
Total asset turnover = $900,000 / $657,000 = 1.37.
Question 22:
2A2-LS17

LMN Corporation has return on common equity of 7% and return on total assets of 10%.
Which of the following statements is true?
The financial leverage index is 1.43, which is considered a good value.
The financial leverage index is 1.43, which is considered a poor value.
The financial leverage index is 0.7, which is considered a good value.
The financial leverage index is 0.7, which is considered a poor value.

The formula for the financial leverage index is return on common equity divided by return on
total assets. The financial leverage index for LMN Corporation is 0.7 (7%/10%). Any value
above 1 is considered good; an index value below 1 is considered poor.
Question 23:
2A2-LS36

ABC Company has revenues of $5,000,000, with a net profit margin of 15%. Its operating
cash flow is $1,300,000, its total assets are $6,000,000 (with current assets of $1,500,000),
and its total liabilities are $2,500,000 (with current liabilities of $500,000). This leaves equity of
$3,500,000, of which $500,000 is preferred shareholders' equity. ABC Company has
1,000,000 shares of common stock outstanding. What is ABC Company's operating cash flow
to income ratio?
0.26.
0.58.
0.22.
1.73.

The operating cash flow to income ratio is calculated by dividing operating cash flow by
income. Net profit can be computed as $5 million × 15% or $750,000. The operating cash flow
to income ratio is 1.73 ($1,300,000 operating cash flow ÷ $750,000 net income).
Question 24:
2A2-AT31

Residco Inc. expects net income of $800,000 for the next fiscal year. Its targeted and current
capital structure is 40% debt and 60% common equity. The director of capital budgeting has
determined that the optimal capital spending for next year is $1.2 million. If Residco follows a
strict residual dividend policy, what is the expected dividend payout ratio for next year?
90%.
10%.
67%.
40%.

The dividend payout ratio is the ratio of dividends to net income in a period. Residco plans to
invest $1,200,000 next year. Given a targeted capital structure of 40% debt and 60% equity,
the $1,200,000 would be financed $480,000 (40%) by debt and $720,000 (60%) by equity
from earnings. Given the projected earnings of $800,000, $80,000 would be left for dividends
($800,000 net income - $720,000 reinvested = $80,000 remaining to be paid as dividends).
Dividend payout ratio = dividends / net income
Residco's dividend payout ratio = $80,000 / $800,000 = 0.1, or 10%.
Question 25:
2A2-CQ36

The following information concerning Arnold Company's common stock was included in the
company's financial reports for the last two years.

Arnold's dividend yield in Year 2:
has declined compared to Year 1.
has increased compared to Year 1.
is the same as Year 1.

is indicative of the company's failure to provide a positive return to the investors.

The dividend yield on common stock is calculated as:
Dividend yield on common stock = (annual dividend per common share) / (market price of
common stock)
Dividend yield, year 1 = $1 / $50 = 0.02, or 2.0%
Dividend yield, year 2 = $1 / $60 = 0.0167, or 1.67%
Therefore, the dividend yield in year 2 has declined compared to year 1.
Question 26:
2A2-LS62

A steady drop in a firm's price/earnings (P/Es) ratio could indicate that:
*Source: Retired ICMA CMA Exam Questions.

both earnings per share and the market price of the stock are rising.
earnings per share has been steadily decreasing.
earnings per share has been increasing while the market price of the stock has held steady.
the market price of the stock has been steadily rising.

The P/Es ratio is the price of the stock divided by the earnings per share.
Question 27:
2A2-CQ31

At year-end, Appleseed Company reported net income of $588,000. The company has 10,000
shares of $100 par value, 6% preferred stock and 120,000 shares of $10 par value common
stock outstanding and 5,000 shares of common stock in treasury. There are no dividend
payments in arrears, and the market price per common share at the end of the year was $40.
Appleseed's price/earnings (P/E) ratio is:
8.16.
8.50.
9.47.
9.09.

The P/E ratio is calculated by taking the market price per share and dividing it by the earnings
per share.
P/E ratio = (market price per share) / (earnings per share)
Earnings per share (EPS) = (net income − preferred stock dividends) / (weighted average
number of shares of common stock outstanding

EPS = ($588,000 − (0.06 × $100))(10,000 shares)) / 120,000 shares
EPS = ($588,000 − $60,000) / 120,000 shares = $528,000 / 120,000 shares = $4.40
P/E ratio = $40 / $4.40 = 9.09.
Question 28:
2A2-CQ22

Grand Savings Bank has received loan applications from three companies in the plastics
manufacturing business and currently has the funds to grant only one of these requests.
Specific data, shown below, has been selected from these applications for review and
comparison with industry averages.

Based on the information above, select the strategy that should be the most beneficial to
Grand Savings.
Grand should not grant any loans as none of these companies represents a good credit risk.
Grant the loan to Springfield as all the company's data approximate the industry average.
Grant the loan to Reston as both the debt/equity (D/E) ratio and degree of financial leverage (DFL)
are below the industry average.
Grant the loan to Valley as the company has the highest net profit margin and degree of financial
leverage.

The D/E ratio and the DFL both measure an organization's risk. The lower the D/E ratio, the
lower the risk. Similarly, the lower the DFL, the lower the risk. In the case of Reston, both the
D/E ratio and DFL measures are below the industry averages. Therefore, it is the least risky of
the three choices.
Question 29:
2A2-CQ35

Mayson Company reported net income of $350,000 for last year. The company had 100,000
shares of $10 par value common stock outstanding and 5,000 shares of common stock in
treasury during the year. Mayson declared and paid $1 per share dividends on common stock.
The market price per common share at the end of last year was $30. The company's dividend
yield for the year was:
30.03%.

28.57%.
11.11%
3.33%.

The dividend yield on common stock is calculated as:
Dividend yield on common stock = (annual dividend per common share) / (market price of
common stock)
Therefore, the dividend yield = $1 / $30 = 0.0333, or 3.33%.
Question 30:
2A2-CQ39

Zubin Corporation experiences a decrease in sales and the cost of goods sold, an increase in
accounts receivable, and no change in inventory. If all else is held constant, what is the total
effect of these changes on the receivables turnover and inventory ratios?
*Source: Retired ICMA CMA Exam Questions.

Inventory Turnover: Increased; Receivables Turnover: Increased.
Inventory Turnover: Decreased; Receivables Turnover: Increased.
Inventory Turnover: Increased; Receivables Turnover: Decreased.
Inventory Turnover: Decreased; Receivables Turnover: Decreased.

The inventory turnover ratio is cost of goods sold (COGS) divided by average inventory. The
accounts receivables turnover is net sales on credit divided by average receivables. Since
sales and COGS decreased, accounts receivable increased, and inventory remained
constant, both ratios would decrease.
Question 1:
2A2-LS04

A firm's total assets are $10,000,000, its total liabilities are $4,000,000, its current assets are
$2,800,000 ($600,000 cash, $1,000,000 money market investments, $700,000 inventory, and
$500,000 receivables), its current liabilities are $900,000 ($400,000 accounts payable,
$500,000 notes payable), and its equity is $6,000,000. Its operating cash flow is $1,500,000.
What is the firm's cash flow ratio?
1.5.
3.75.
1.67.
3.

Cash flow ratio = Operating Cash Flow / Current Liabilities = $1,500,000 / $900,000 = 1.67.

Question 2:
2A2-AT37

Assuming that Lisa Inc.'s net income for Year 2 was $35,000 and there are no preferred stock
dividends in arrears, Lisa's return on common equity (ROE) for Year 2 was:

12.4%.
10.6%.
7.8%.
10.9%.

ROE is calculated as:
ROE = (net income − preferred stock dividends) / (average common shareholders' equity)
The average common shareholders' equity is calculated by taking the beginning balance of
common shareholders' equity less the preferred stock equity, plus the ending balance of
common shareholders' equity less the preferred stock equity, and dividing that total by 2.
Dividing by two calculates the average for the period.
Average common shareholders' equity = [(beginning balance common shareholders' equity −
preferred stock equity) + (ending balance common shareholders' equity − preferred stock
equity)] / 2
Lisa's average common shareholders' equity = [($390 - $100) + ($375 − $100)] / 2 = ($565) / 2
= $282.50
Lisa's preferred stock dividend is 5% of $100, or 0.05($100) = $5. Net income is $35.
Therefore, ROE = ($35 − $5) / ($282.50) = $30/$282.50 = 0.106, or 10.6%.
Question 3:
2A2-LS18

During the current year, Beverly Industries reports an inventory turnover of 10 times, a gross
profit margin of 30%, a net profit margin of 4%, and average inventory of $21,000. What are
net sales for the year?
$2,800,000.
$700,000.
$210,000.
$300,000.

Since we know that average inventory is $21,000 and the inventory turnover rate is 10X, we
can find that cost of goods sold (COGS) is $210,000. Since the gross profit margin is 30%, the
firm's COGS must be 70% of net sales. Therefore, net sales must be $300,000.
Question 4:
2A2-CQ30

Devlin Inc. has 250,000 shares of $10 par value common stock outstanding. For the current
year, Devlin paid a cash dividend of $3.50 per share and had earnings per share of $4.80.
The market price of Devlin's stock is $34 per share. Devlin's price/earnings (P/E) ratio is:
9.71.
2.08.
2.85.
7.08.

The P/E ratio is calculated by taking the market price per share and dividing it by the earnings
per share.
P/E ratio = (market price per share) / (earnings per share)
P/ E ratio = $34 / $4.80 = 7.08.
Question 5:
2A2-AT34

Devlin Company's times interest earned for the year ended May 31, Year 2, was:
12.25 times.
11.25 times.
6.75 times.
18.75 times.

The number of times interest is earned (interest coverage) measures a firm's ability to pay
long-term debt using operating income. Times interest earned is calculated by dividing
operating income by the interest expense. Operating income is calculated by taking net sales
and subtracting both cost of goods sold and operating expenses (selling, general, and
administrative).
For Devlin, operating income is calculated by taking income before taxes plus interest
expense ($90 + $8 = $98).
Devlin's times interest earned is calculated as:
Times interest earned = operating income / interest expense
Times interest earned = $98 / $8 = 12.25.
Question 6:
2A2-CQ09

Maydale Inc.'s financial statements show the following information.

Maydale's accounts receivable turnover ratio is:

11.25.
0.10.
9.
10.

The accounts receivable turnover per year is calculated as:
Accounts receivable turnover per year = (Net credit sales for year) / (average accounts
receivable balance for the year)
Average accounts receivable balance for the year = (beginning balance + ending balance) / 2
Average accounts receivable balance for the year = ($320,000 + $400,000) / 2 = $720,000 / 2
= $360,000
Turnover per year = $3,600,000 / $360,000 = 10 times.
Question 7:
2A2-AT17

Which of the following would decrease a firm's working capital?
The firm purchases a fixed asset, paying 20% of the purchase price in cash and signing a long-term
note for the remaining balance.
The firm purchases inventory on account.
The firm receives payment on an outstanding accounts receivable.
The firm writes off a bad account (receivable). The firm uses the allowance method for bad debts.

Working capital is calculated by taking current assets less current liabilities. Current assets
consist of cash, short-term marketable securities, accounts receivable, short-term notes
receivable, inventories, and prepaid expenses. Current liabilities are those due within one
year, or one business cycle, whichever is longer. Current liabilities consist of accounts
payable, accrued expenses payable, interest payable, taxes payable, short-term notes
payable, and the current portion of long-term debt.
Working capital decreases when current assets decrease by more than current liabilities or
increase by less than current liabilities. The purchase of a fixed (long-term) asset partly by
cash and partly by signing a long-term note will decrease current assets without affecting
current liabilities. Therefore, working capital will decrease.
Question 8:
2A2-LS01

Consider the following transactions:

I. A firm receives cash on account.
II. A firm sells goods on account (cost of goods sold (COGS) is less than sales price).
III. A firm makes a payment on account.
IV. A firm purchases inventory on account.
If a firm has a current ratio greater than one, which of the transactions above would cause its
current ratio to increase?
I, II, III, and IV.
I, II, and III.
II, III, and IV.
II and III.

The current ratio is defined as current assets divided by current liabilities. Examine each
transaction for its effect on either current assets or current liabilities:
I. Debit to cash, credit to accounts receivable; the net result is no change in either current
assets or current liabilities. Therefore, there is no change to the current ratio.
II. Debit to accounts receivable, credit to sales for an amount greater than a debit to COGS
and a credit to inventory. The result will be an increase in current assets. Therefore, there is
an increase in the current ratio.
III. Debit to accounts payable, credit to cash. The result of this transaction is an equal
decrease in current assets and current liabilities. Given that the current ratio is greater than 1,
this transaction will result in an increase in the current ratio.
IV. Debit to inventory and credit to accounts payable. The result of this transaction is an equal
increase in current assets and current liabilities. Given that the current ratio is greater than 1,
this transaction will result in a decrease in the current ratio.
Question 9:
2A2-LS02

Consider the following transactions:
I. A firm receives cash on account.
II. A firm sells goods on account (cost of goods sold [COGS] is less than sales price).
III. A firm makes a payment on account.
IV. A firm purchases inventory on account.
If a firm has a quick ratio greater than one, which of the transactions above would cause its
quick ratio to decrease?
I, II, and III.
I and IV.

None of these choices.
IV only.

br/> The quick ratio is defined as current assets less inventory divided by current liabilities.
Examine each transaction for its effect on either current assets or current liabilities:
I. Debit to cash, credit to accounts receivable; the net result is no change in either current
assets or current liabilities. Therefore, there is no change to the quick ratio.
II. Debit to accounts receivable, credit to sales for an amount greater than a debit to COGS
and a credit to inventory. The result will be an increase in current assets less inventory.
Therefore, there is an increase in the quick ratio.
III. Debit to accounts payable, credit to cash. The result of this transaction is an equal
decrease in current assets and current liabilities. Given that the quick ratio is greater than 1,
this transaction will result in an increase in the quick ratio.
IV. Debit to inventory and credit to accounts payable. The result of this transaction is no
change in current assets less inventory and an increase in current liabilities. This transaction
will decrease the quick ratio.
Question 10:
2A2-CQ32

Archer Inc. has 500,000 shares of $10 par value common stock outstanding. For the current
year, Archer paid a cash dividend of $4.00 per share and had earnings per share of $3.20.
The market price of Archer's stock is $36 per share. The average price/earnings (P/E) ratio for
Archer's industry is 14.00. When compared to the industry average, Archer's stock appears to
be:
undervalued by approximately 20%.
overvalued by approximately 10%.
undervalued by approximately 25%.
overvalued by approximately 25%.

The P/E ratio is calculated by taking the market price per share and dividing it by the earnings
per share.
P/E ratio = (market price per share) / (earnings per share)
If Archer's stock tracked with the market, then its stock price would be equal to its EPS
multiplied by the industry P/E ratio, as:
Archer's stock price = ($3.20 per share)(14) = $44.80
Since the stock is selling at only $36, it is undervalued by approximately 20%, which is
calculated as: ($44.80 − $36) / $44.80 = $8.80 / $44.80 = 0.196, which rounds to 20%.
Question 11:

2A2-LS32

Selected data from KJ, Inc., financial statements for the end of the current year are:

What is ending inventory for the current year?
$210,000.
$600,000.
$90,000.
$390,000.

If current liabilities are $300,000 and the current ratio is 2.0, then total current assets must be
$600,000. If current liabilities are $300,000 and the quick ratio is 1.3, then total current assets
less inventory must be 390,000. The difference between $600,000 and $390,000 represents
inventory, which is $210,000.
Question 12:
2A2-AT12

Assume the following information for Ramer Company, Matson Company and for their
common industry for a recent year.

The attitudes of both Ramer and Matson concerning risk are best explained by the:
debt/equity ratio and times interest earned.
current ratio and earnings per share.
return on investment and dividend payout ratio.
current ratio, accounts receivable turnover, and inventory turnover.

Risk is a function of financial leverage. Financial leverage is measured by the debt/equity ratio
and the number of times interest is earned (interest coverage).
Question 13:
2A2-AT11

Assume the following information for Ramer Company, Matson Company and for their
common industry for a recent year.

Which is correct if both companies have the same total assets and the same sales?
Matson has a shorter operating cycle than Ramer.
Ramer has fewer current liabilities than Matson.
Ramer has more cash than Matson.
Matson is more effectively using financial leverage.

The operating cycle is the time period from the acquisition of raw materials to the collection of
the cash from the sale of the goods produced from the raw materials. It is the combination of
the number of days on hand for inventory and the accounts receivable collection period.
The days on hand is calculated as 365 days divided by the inventory turnover. The accounts
receivable collection period is calculated as 365 days divided by the accounts receivable
turnover.
Ramer's and Matson's operating cycles are calculated as:
Operating cycle = (365 days / company's inventory turnover) + (365 days / company's
accounts receivable turnover)
Ramer's operating cycle = (365 days / 6.2 days) + (365 days / 5 days) = 53.9 days + 73 days
= 131.9 days
Matson's operating cycle = (365 days / 8 days) + (365 days / 8.1 days) = 45.6 days + 45.1
days = 90.7 days
From these calculations, it is evident that Matson has a shorter operating cycle than Ramer.

Question 14:
2A2-LS59

The use of debt in the capital structure of a firm:
*Source: Retired ICMA CMA Exam Questions.

increases its operating leverage.
decreases its financial leverage.
decreases its operating leverage.
increases its financial leverage.

Financial leverage is the use of debt (fixed cost funds) to increase returns to owners
(stockholders).
Question 15:
2A2-LS15

In which of the following circumstances would financial leverage be likely to increase?
Firm issues bonds to repurchase some of its own common stock.
Firm purchases assets with cash.
Firm signs a contract to rent a new manufacturing site.
Firm accepts a large order from a new customer. Excess capacity exists to fill the order.

Financial leverage relates to the fixed financing costs of a firm. Of the choices given, issuing
new bonds is the only one that affects fixed financing costs.
Question 16:
2A2-LS03

A firm's total assets are $10,000,000, its total liabilities are $4,000,000, its current assets are
$2,800,000 million ($600,000 cash, $1,000,000 money market investments, $700,000
inventory, and $500,000 receivables), its current liabilities are $900,000 ($400,000 accounts
payable, $500,000 notes payable), and its equity is $6,000,000. What is the firm's cash ratio?
2.5.
3.11.
1.5.
1.78.

The cash ratio is cash and cash equivalents and marketable securities divided by current
liabilities. ($600,000 cash + $1,000,000 money market) ÷ $900,000 = $1,600,000/$900,000 =
1.78.
Question 17:
2A2-AT27

CPZ Enterprises had the following account information.

The company has an operating cycle of five months.
What is the company's acid test (quick) ratio?
1.68 to 1.
2.14 to 1.
5.29 to 1.
2.31 to 1.

The acid-test ratio (also known as the quick ratio) is the ratio of quick assets to current
liabilities. Quick assets are those easily converted to cash without significant loss. The quick
assets are cash, short-term investments (marketable securities), and net receivables. Current
liabilities are those due within one year, or one business cycle, whichever is longer. Current
liabilities consist of accounts payable, accrued expenses payable, interest expense payable,
taxes payable, short-term notes payable, and the current portion of long-term debt.
CPZ's acid-test ratio is calculated as:
Acid-test ratio = (cash + marketable securities + net receivables) / current liabilities
Acid-test ratio = ($100,000 cash + $200,000 accounts receivable) / ($80,000 accounts
payable + $10,000 interest payable + $50,000 notes payable (due in six months))
Acid-test ratio = ($300,000) / ($140,000) = ratio of 2.14 to 1.
Question 18:
2A2-LS20

A company has $500,000 in current assets and $1,500,000 in fixed assets. Its paid-in capital
is $100,000 and retained earnings are $1,400,000. 40% of the debt is considered to be
current. What is its long-term debt-to-equity capital ratio?
5.
0.2.
0.21.
0.53.

The long-term debt-to-equity capital ratio is calculated by dividing long-term debt by
shareholders' equity. The company's total shareholders' equity is $1,500,000 ($100,000 paidin capital + $1,400,000 retained earnings). Its debt is $500,000 (assets − equity = $2 million −
$1,500,000). If current debt is 40% of debt, then long-term debt is 60% of $500,000, or
$300,000. Therefore, its long-term debt-to-equity capital ratio is 0.2 ($300,000/$1,500,000).
Question 19:
2A2-LS58

Firms with high degrees of financial leverage would be best characterized as having:
*Source: Retired ICMA CMA Exam Questions.

high fixed-charge coverage.
zero coupon bonds in their capital structures.
high debt-to-equity ratios.
low current ratios.

Financial leverage is the use of debt (fixed cost funds) to increase returns to owners
(stockholders). A high degree of financial leverage means the benefits from tax-deductibility of
interest (from additional debt) is more than offset by the increase in the payments to repay the
debt.
Question 20:
2A2-CQ02

Birch Products Inc. has the following current assets.

If Birch's current liabilities are $1,300,000, the firm's:
current ratio will decrease if a payment of $100,000 cash is used to pay $100,000 of accounts
payable.
current ratio will not change if a payment of $100,000 cash is used to pay $100,000 of accounts
payable.
quick ratio will not change if a payment of $100,000 cash is used to purchase inventory.
quick ratio will decrease if a payment of $100,000 cash is used to purchase inventory.

The quick ratio is calculated as:
Quick ratio = (cash + marketable securities + receivables) / (current liabilities).

Question 21:
2A2-CQ41

Douglas Company purchased 10,000 shares of its common stock at the beginning of the year
for cash. This transaction will affect all of the following except the:
*Source: Retired ICMA CMA Exam Questions.

net profit margin.
earnings per share.
current ratio.
debt-to-equity ratio.

The issuance of stock has no effect on profit margin (net income divided by sales). It would
increase equity, decreasing the debt/equity ratio. The increased number of shares outstanding
would decrease earnings per share (net income divided by shares). The cash received would
increase cash, a current asset, and the current ratio (current assets divided by current
liabilities).
Question 22:
2A2-AT40

The average collection period for a firm measures the number of days:
beyond the end of the credit period before a typical customer payment is received.
before a typical account becomes delinquent.
after a typical credit sale is made until the firm receives the payment.
it takes a typical check to "clear" through the banking system.

The accounts receivable collection period is the average length of time between when a credit
sale is made and when it is collected. It is calculated by dividing the number of days in a year
(365) by the accounts receivable turnover. The accounts receivable turnover is the annual
sales divided by the average accounts receivable balance for the year.
Question 23:
2A2-LS25

ROK Company has $900,000 in current assets and $2,100,000 in fixed assets, including
$300,000 goodwill and patents. Its paid-in capital is $100,000 and retained earnings are
$1,900,000. What is its total liabilities to net tangible assets ratio?
0.37.
2.57.
0.47.
0.56.

The total liabilities to net tangible assets ratio is calculated by dividing total liabilities by net
tangible assets. Total liabilities are 1,000,000 (total assets less equity = $3 million − $2
million). Net tangible assets for ROK Company are $2,700,000 ($900,000 current assets +
$2,100,000 fixed assets − $300,000 intangible assets). The total liabilities to net tangible
assets ratio is 0.37 ($1,000,000 total liabilities ÷ $2,700,000 net tangible assets).
Question 24:
2A2-LS13

ABC Company has $1 million in assets and a debt-to-equity ratio of 66 2/3%. If ABC
purchases an asset worth $200,000 with debt, its new debt-to-equity ratio will be:
100%.
80%.
75%.
66 2/3%.

The debt-to-equity ratio is defined as total debt divided by total equity. Given that the current
debt-to-equity ratio is 66 2/3% = 2/3, then debt must currently be $400,000 and equity must be
$600,000. After the transaction occurs, debt will be $600,000 and equity will not change.
Therefore, the new debt-to-equity ratio must be 100%.
Question 25:
2A2-LS39

ABW Corporation's net income is $1,000,000, and it has 1,000,000 shares of common stock
outstanding. It also has 50,000 shares of 10%, $20 par preferred stock outstanding. What is
ABW Corporation's earnings per share?
$0.90.
$1.
$2.
$20.

Earnings per share (EPS) is calculated by dividing the earnings available to common
shareholders by the number of common shares outstanding. Remember that earnings
available to common shareholders is net income minus preferred dividends. Preferred
dividends are computed as 50,000 shares × 10% of $20 par or $100,000. Earnings available
to common shareholders are $900,000 ($1,000,000 net income − $100,000 preferred
dividend). EPS is $0.90 ($900,000 ÷ 1,000,000 shares of common stock outstanding).
Question 26:
2A2-CQ05

Selected financial data for Boyd Corporation are shown below.

Boyd's net income for the year was $96,000. Boyd's current ratio at the end of the year is:
2.71.
1.55.
1.71.
2.97.

The current ratio is defined as follows:
Current ratio = (current assets) / (current liabilities)
Current assets include cash, receivables, trading securities, and inventories. Current liabilities
include accounts payable and accrued liabilities.
Current ratio = ($62,000 + $47,000 + $35,000 + $138,000) / ($84,000 + $11,000)
Current ratio = $282,000 / $95,000 = 2.97.
Question 27:
2A2-CQ43

Colonie Inc. expects to report net income of at least $10 million annually for the foreseeable
future. Colonie could increase its return on equity by taking which of the following actions with
respect to its inventory turnover and the use of equity financing?
Inventory Turnover Use of Equity Financing:
*Source: Retired ICMA CMA Exam Questions.

Inventory Turnover: Decrease; Use of Equity Financing: Decrease.
Inventory Turnover: Increase; Use of Equity Financing: Decrease.
Inventory Turnover: Increase; Use of Equity Financing: Increase.
Inventory Turnover: Decrease; Use of Equity Financing: Increase.

To increase its return on equity (net income divided by average common shareholders' equity.
Colonie could increase its inventory turnover by liquidating inventory and increasing net
income. Colonie could also increase the return by decreasing the use of equity financing. The
decrease would lower the denominator in the formula without affecting the numerator.
Question 28:
2A2-LS52

Garstka Auto Parts must increase its acid test ratio above the current 0.9 level in order to
comply with the terms of a loan agreement. Which one of the following actions is most likely to
produce the desired results?
*Source: Retired ICMA CMA Exam Questions.

Expediting collection of accounts receivable.
Selling auto parts on account.
Purchasing marketable securities for cash.
Making a payment to trade accounts payable.

The acid test ratio is calculated as current assets less inventories and prepayments divided by
total current liabilities. Selling auto parts on account would increase the acid test ratio
increasing the accounts receivable without changing current liabilities.
Question 29:
2A2-AT45

Clauson Inc. grants credit terms of 1/15, net 30 and projects gross sales for next year of
$2,000,000. The credit manager estimates that 40% of their customers pay on the discount
date, 40% on the net due date, and 20% pay 15 days after the net due date. Assuming
uniform sales and a 365-day year, what is the projected days sales outstanding (rounded to
the nearest whole day)?
27 days.
30 days.
20 days.
24 days.

The average days sales outstanding is 27. 40% are outstanding for 15 days, 40% are
outstanding for 30 days, and 20% are outstanding for 45 days, which works out to: 0.4(15) +
0.4(30) + 0.2(45) = 6 + 12 + 9 = 27 days.
Question 30:
2A2-LS10

Which of the following ratios is NOT considered a solvency measure?
Times interest earned.
Debt-to-equity ratio.

Current ratio.
Financial leverage index.

The current ratio is considered a liquidity ratio and not a solvency ratio. The other ratios listed
are considered to be solvency ratios.
Question 1:
2A2-CQ25

The following information has been derived from the financial statements of Boutwell
Company.

The company's debt-to-equity ratio is:
0.50 to 1.
0.33 to 1.
0.37 to 1.
0.13 to 1.

The debt/equity (D/E) ratio is the ratio of total liabilities to total stock equity.
Total liabilities = long-term liabilities + current liabilities
Current ratio = (current assets) / (current liabilities)
Current assets are given as $640,000; Current ratio is given as 3.2. Input these values, then
rearrange the equation to solve for current liabilities.
3.2 = $640,000 / (current liabilities)
Current liabilities = $640,000 / 3.2 = $200,000
Total liabilities = $130,000 of long-term liabilities + $200,000 of current liabilities = $330,000
Total stock equity = total assets − total liabilities
Total stock equity = $990,000 − $330,000 = $660,000
D/E ratio = (total liabilities) / (total stock equity)
D/E ratio = $330,000 / $660,000 = 0.5 to 1.
Question 2:
2A2-AT07

The Statement of Financial Position for King Products Corporation for the fiscal years ended
June 30, Year 2, and June 30, Year 1, is presented below. Net sales and cost of goods sold
for the year ended June 30, Year 2, were $600,000 and $440,000, respectively.

King Products Corporation's quick (acid test) ratio at June 30, Year 2, was:
1.12 to 1.
2.00 to 1.

1.81 to 1.
0.63 to 1.

The acid-test ratio (also known as the quick ratio) is the ratio of quick assets to current
liabilities. Quick assets are those easily converted to cash without significant loss. The quick
assets are cash, short-term investments (marketable securities), and net receivables.
King's acid-test ratio for Year 2 is calculated as:
Acid-test ratio = (cash + marketable securities + net receivables) / current liabilities
Acid-test ratio = ($60 cash + $40 marketable securities + $90 net accounts receivable) / ($170
current liabilities)
Acid-test ratio = $190 / $170 = ratio of 1.12 to 1.
Question 3:
2A2-CQ26

The interest expense for a company is equal to its earnings before interest and taxes (EBIT).
The company's tax rate is 40%. The company's times-interest earned ratio is equal to:
1.0.
2.0.
0.6.
1.2.

The times-interest earned ratio (interest coverage) is the ratio of EBIT to interest expense. If
EBIT is equal to interest expense, then the ratio will equal 1.
Question 4:
2A2-LS49

On a manufacturing company's income statement, sales are $50 million and cost of goods
manufactured equals $25 million. If beginning and ending finished goods inventory are $3
million and $4 million, respectively, which of the following statements is true?
Gross profit equals $26 million.
Gross profit equals $25 million.
Contribution margin equals $25 million.
Gross profit equals $24 million.

The gross profit is sales minus the cost of goods sold. Cost of goods sold is beginning
finished goods inventory plus cost of goods manufactured minus ending finished goods
inventory ($3 + 25 − 4 = $24 million). Therefore, gross profit is $50 million − $24 million = $26
million.
Question 5:
2A2-LS35

ABC Company has revenues of $5,000,000, with net income of $750,000. Its total assets are
$6,000,000 (with current assets of $1,500,000), and its total liabilities are $2,500,000 (with
current liabilities of $500,000). This leaves equity of $3,500,000, of which $500,000 is
preferred shareholders' equity. ABC Company has 1,000,000 shares of common stock
outstanding. What is ABC Company's book value per common share?
$3.
$3.50.
$1.
$2.

Book value per share is calculated by dividing common shareholders' equity by number of
shares outstanding. Common shareholders' equity is $3,000,000 ($3,500,000 total equity −
$500,000 preferred shareholders' equity). Book value per share is $3 ($3,000,000 common
equity ÷ 1,000,000 shares).
Question 6:
2A2-AT14

Birch Corporation had net income for the year of $101,504 and a simple capital structure
consisting of the following common shares outstanding.

Birch Corporation's earnings per share (EPS) (rounded to the nearest cent) was:
$3.44.
$3.26.
$3.20.
$4.23.

EPS = (net income − the preferred stock dividend) / (weighted average number of common
stock shares outstanding)
Birch has no preferred stock.
Therefore, EPS = (net income) / (weighted average number of common stock shares
outstanding)
The weighted average number of common stock shares outstanding (W) is calculated as
follows:

W = (24,000 shares for 2 months) + (29,400 shares for 4 months) + (36,000 shares for 5
months) + (35,040 shares for 1 month)
W = (2/12)(24,000) + (4/12)(29,400) + (5/12)(36,000) + (1/12)(35,040) = 4,000 + 9,800 +
15,000 + 2,940 = 31,720 shares.
EPS = ($101,504) / (31,720 shares) = $3.20 per share.
Question 7:
2A2-LS43

Which of the following are limitations of ratio analysis?
I. Ratios must have something to compare to in order to have meaning.
II. There must be a relationship between the two accounts used in the ratio.
III. The analyst must be aware of the accounting principles employed by the business.
IV. The analyst must be certified by the PAA.
I only.
I and III only.
I, II, III, and IV.
I, II, and III only.

Whether or not analysts must be certified, choice IV does not represent a limitation of ratios
themselves.
Question 8:
2A2-AT06

The Statement of Financial Position for King Products Corporation for the fiscal years ended
June 30, Year 2, and June 30, Year 1, is presented below. Net sales and cost of goods sold
for the year ended June 30, Year 2, were $600,000 and $440,000, respectively.

King Products Corporation's average collection period for the fiscal year ended June 30, Year
2, using a 365-day year, was:
46 days.
36 days.
54 days.
61 days.

The accounts receivable collection period is calculated by taking the number of days in a year
(in this case, that would be 365 days) and dividing that number by the accounts receivable
turnover.
Accounts receivable turnover is calculated by dividing net sales by the average accounts
receivable balance.
The average accounts receivable, if not given, is calculated by adding together the beginning
and ending balances and dividing by 2. Note that the beginning accounts receivable balance
for Year 2 is the same as the ending accounts receivable balance for Year 1.
King's average accounts receivable, accounts receivable turnover, and average collection
period are calculated as:
King's average accounts receivable for Year 2 = ($90 +$60)/2 = $160/2 = $75.
Accounts receivable turnover = net sales / average accounts receivable balance
King's accounts receivable turnover for year 2 = $600 / $75 = 8 times
King's average collection period for year 3 is then calculated by taking the number of days in a
year and dividing it by the accounts receivable turnover.
King's average collection period = 365 days / 8 = 45.6 days, round to 46 days
Question 9:
2A2-AT09

Nelson Industries increased earnings before interest and taxes by 17%. During the same
period, net income after tax increased by 42%. The degree of financial leverage (DFL) that
existed during the year is:
2.47.
1.70.
1.68.
4.20.

The DFL is defined as the percent change in net income after tax given a percent change in
operating income (earnings before interest and taxes, or EBIT).
Nelson's degree of financial leverage is calculated as:
DFL = (% change in net income after tax) / (% change in operating income, or EBIT)
Nelson's DFL = (42%) / (17%) = 2.47.
Question 10:
2A2-AT43

The Dawson Corporation projects the following for the year.

The expected common stock dividend per share for Dawson Corporation for the year is:
$2.10.
$1.80.
$2.70.
$3.90.

The expected common stock dividend for the year is $2.10 per share, which is calculated as:
The dividend payout ratio is 30% of earnings to common shareholders. Earnings to common
shareholders is $14 million.
The dividend per share is ($14,000,000)(0.3)/(2,000,000 shares) = $ 4,200,000/2,000,000
shares = $2.10 per share.
The $14,000,000 in earnings to common shareholders (E to CS) is calculated as follows:
EBIT = earnings before interest and taxes
E to CS = [(EBIT − Interest)(1 − Tax Rate)] − (Preferred Stock Dividend)
E to CS = [($35,000,000 − $5,000,000)(1 - 0.4)] − ($4,000,000)
= [$30,000,000(0.6)] − ($4,000,000)
= $18,000,000 − $4,000,000 = $14,000,000.
Question 11:
2A2-LS60

Which one of the following statements concerning the effects of leverage on earnings before
interest and taxes (EBIT) and earnings per share (EPS) is correct?
*Source: Retired ICMA CMA Exam Questions.

Financial leverage affects both EPS and EBIT, while operating leverage only effects EBIT.
If Firm A has a higher degree of operating leverage than Firm B, and Firm A offsets this by using less
financial leverage, then both firms will have the same variability in EBIT.
For a firm using debt financing, a decrease in EBIT will result in a proportionally larger decrease in
EPS.

A decrease in the financial leverage of a firm will increase the beta value of the firm.

Debt financing creates financial leverage (the percent change in EPS given a percent change
in EBIT) which is always greater than one. Therefore, a change in EBIT causes a
proportionally larger change in EPS.
Question 12:
2A2-AT05

The Statement of Financial Position for King Products Corporation for the fiscal years ended
June 30, Year 2, and June 30, Year 1, is presented below. Net sales and cost of goods sold
for the year ended June 30, Year 2, were $600,000 and $440,000, respectively.

King Products Corporation's accounts receivable turnover for this period was:
8.
10.
6.7.
5.9.

Accounts receivable turnover is calculated by dividing net sales by the average accounts
receivable balance. The average accounts receivable, if not given, is calculated by adding
together the beginning and ending balances and dividing by 2. Note that the beginning
accounts receivable balance for year 2 is the same as the ending accounts receivable
balance for year 1.
King's average accounts receivable balance and accounts receivable turnover are calculated
as follows:
King's average accounts receivable for year 2 = ($90 +$60)/2 = $160/2 = $75
Accounts receivable turnover = net sales / average accounts receivable balance
King's accounts receivable turnover for year 2 = $600 / $75 = 8 times.
Question 13:
2A2-CQ44

Donovan Corporation recently declared and issued a 50% stock dividend. This transaction will
reduce the company's:
*Source: Retired ICMA CMA Exam Questions.

book value per common share.
return on operating assets.
debt-to-equity ratio.
current ratio.

Book value per common share (common shareholders' equity divided by number of common
shares outstanding). A stock dividend would increase the number of shares outstanding
without affecting the equity.
Question 14:
2A2-AT41

A firm's financial risk is a function of how it manages and maintains its debt. Which one of the
following sets of ratios characterizes the firm with the greatest amount of financial risk?
high debt-to-equity ratio, low interest coverage ratio, volatile return on equity.
low debt-to-equity ratio, low interest coverage ratio, volatile return on equity.
high debt-to-equity ratio, high interest coverage ratio, volatile return on equity.
low debt-to-equity ratio, high interest coverage ratio, stable return on equity.

Financial risk is represented by the variability in expected returns, and a volatile return on
equity is risky. An increase in the debt-to-equity ratio increases risk by the increased debt
load. A decrease in the interest coverage ratio increases risk by decreasing the organization's
ability to service its debt.

Question 15:
2A2-AT38

The two financial statements that are most important for assessing a firm's liquidity are the:
balance sheet and retained earnings statement.
income statement and statement of cash flows.
income statement and balance sheet.
balance sheet and statement of cash flows.

Liquidity is typically measured by cash flow from operations in the statement of cash flows, by
the current and quick ratios taken from the balance sheet, and by the operating cash flow to
current liabilities ratio which uses both statements. The current ratio equals current assets
divided by current liabilities. The quick ratio (acid test) equals the “quick” assets divided by the
current liabilities. The “quick” assets are cash, cash equivalents, short-term investments, and
receivables.
Question 16:
2A2-LS30

MON Corporation has total assets of $2,000,000. Its net profit margin is 5% with revenues of
$13,050,000, and its preferred dividends are $25,000. What is MON Corporation's return on
total assets ratio?
0.36.
0.33.
0.48.
0.28.

The return on total assets ratio is calculated by dividing net income by total assets. Net
income for MON Corporation is $652,500 ($13,050,000 × 5%). Its return on total assets ratio
is 0.33 ($652,500 net income ÷ $2,000,000 assets).
Question 17:
2A2-CQ03

Shown below are beginning and ending balances for certain of Grimaldi Inc.'s accounts.

Grimaldi's acid test ratio or quick ratio at the end of the year is:
1.15.
1.52.
0.83.
1.02.

The quick ratio is calculated as:
Quick ratio = (current assets − inventories − prepayments) / (current liabilities)
Current assets = (cash + marketable securities + receivables + inventory)
Current Assets = $62,000 + $35,000 + $47,000 + $138,000 = $282,000
Current liabilities = (accounts payable + accrued liabilities)
Current liabilities = $84,000 + $11,000 = $95,000
Quick Ratio = (Current assets − Inventory − Prepayments) / Current Liabilities
Quick Ratio = ($282,000 − $138,000 − $0) / $95,000
Quick ratio = $144,000 / $95,000 = 1.516, which rounds to 1.52.
Question 18:
2A2-LS53

The owner of a chain of grocery stores has bought a large supply of mangoes and paid for the
fruit with cash. This purchase will adversely impact which one of the following?
*Source: Retired ICMA CMA Exam Questions.

Current ratio.
Working capital.
Price earnings ratio.
Quick or acid test ratio.

The acid test ratio is calculated as current assets less inventories and prepayments divided by
total current liabilities. Purchasing inventory and paying cash would adversely impact this ratio
by decreasing cash. The cash purchase of inventory has no effect on current assets and,
therefore, will not affect working capital (current assets − current liabilities) or the current ratio
(current assets divided by current liabilities). Purchases have no effect on the price earnings
ratio (stock price divided by EPS).
Question 19:
2A2-LS55

The acid test ratio shows the ability of a company to pay its current liabilities without having to:
*Source: Retired ICMA CMA Exam Questions.

reduce its cash balance.
liquidate its inventory.
borrow additional funds.
collect its receivables.

The acid test ratio is calculated as current assets less inventories and prepayments divided by
total current liabilities. The acid test ratio shows the ability of a company to pay its current
liabilities without having to liquidate its inventory.
Question 20:
2A2-LS57

If a company has a current ratio of 2.1 and pays off a portion of its accounts payable with
cash, the current ratio will:
*Source: Retired ICMA CMA Exam Questions.

increase.
decrease.
move closer to the quick ratio.
remain unchanged.

The current ratio is calculated as current assets divided by current liabilities. Paying off a
portion of accounts payable with cash will cause the current ratio to increase by decreasing
both current assets and current liabilities by the same amount.
Question 21:
2A2-LS41

ABC Corporation has fully diluted earnings per share of $5, pays a dividend of $1 per share,
and has a current market price of $60 per share. What is ABC Corporation 's dividend payout
ratio?
0.012.
0.2.
0.017.
0.083.

The dividend payout ratio is calculated by dividing dividends per common share by fully
diluted earnings per share. $1 dividend per share ÷ $5 fully diluted earnings per share = 0.2.
Question 22:
2A2-AT15

At the beginning of the fiscal year, June 1, Year 1, Boyd Corporation had 80,000 shares of
common stock outstanding. Also outstanding was $200,000 of 8% convertible bonds that had
been issued at $1,000 par. The bonds were convertible into 20,000 shares of common stock;
however, no bonds were converted during the year. The company's tax rate is 34%, and the

Aa bond interest rate has been 10%. Boyd's net income for the year was $107,000. The fully
diluted earnings per share (EPS) (rounded to the nearest cent) of Boyd common stock for the
fiscal year ended May 31, Year 2, was:
$1.18.
$1.12.
$1.23.
$1.07.

EPS = (net income − the preferred stock dividend)/(weighted average number of common
stock shares outstanding).
Calculating fully diluted EPS requires the adjusting of the numerator and denominator for the
effects of conversions of convertible securities and/or the exercise of options or warrants.
Boyd's basic EPS = (net income) / (80,000 shares) = ($107,000)/(80,000 shares)
Fully diluted EPS requires adjusting the numerator for the after-tax effect of the interest
expense eliminated by the conversion, and adjusting the denominator for the additional
20,000 shares from the conversion. The conversion is assumed to occur at the earliest
possible date; in this case, June 1 of Year 1.
The after-tax effect of the interest expense = (1 − tax rate)(interest expense)
= (1 − 0.34)(0.08)($200,000) = (0.66)($16,000) = $10,560
Boyd's fully diluted EPS = ($107,000 + $10,560) / (80,000 shares + 20,000 shares) =
($117,560)/(100,000 shares) = $1.18 per share.
Question 23:
2A2-LS56

All of the following are included when calculating the acid test ratio except:
*Source: Retired ICMA CMA Exam Questions.

six-month treasury bills.
60-day certificates of deposit.
prepaid insurance.
accounts receivable.

The acid test ratio is calculated as cash plus marketable securities plus accounts receivable
divided by total current liabilities or as current liabilities less inventories and prepayments
divided by current liabilities.
Question 24:

2A2-CQ14

Cornwall Corporation's net accounts receivable were $68,000 and $47,000 at the beginning
and end of the year, respectively. Cornwall's condensed Income Statement is shown below.

Cornwall's average number of days' sales in accounts receivable (using a 365-day year) is:
23 days.
13 days.
8 days.
19 days.

The average number of days in accounts receivable is calculated as:
Average number of days in accounts receivable = (# days in a year) / (the accounts receivable
turnover per year)
Accounts receivable turnover per year = (Net credit sales for year) / (average accounts
receivable balance for the year)
Average accounts receivable balance for the year = (beginning balance + ending balance) / 2
Average accounts receivable balance for the year = ($68,000 + $47,000) / 2 = $115,000 / 2 =
$57,500
Accounts receivable turnover per year = $900,000 / $57,500 = 15.65 times per year
Average number of days in accounts receivable = 365 days / 15.65 times = 23 days.
Question 25:
2A2-CQ11

Garland Corporation's Income Statement for the year just ended is shown below.

Garland's average inventory turnover ratio is:
4.01.
3.82.
6.84.
6.52.

The inventory turnover per year is calculated as:
Inventory turnover per year = (cost of goods sold for year) / (average inventory balance for the
year)
Average inventory balance for the year = (beginning balance + ending balance) / 2
Average inventory balance for the year = ($125,000 + $138,000) / 2 = $263,000 / 2 =
$131,500
Turnover per year = $527,000 / $131,500 = 4.01 times.
Question 26:
2A2-AT13

Assume the following information for Ramer Company, Matson Company and for their
common industry for a recent year.

Some of the ratios and data for Ramer and Matson are affected by income taxes. Assuming
no interperiod income tax allocation, which one of the following items would be directly
affected by income taxes for the period?
times interest earned and current ratio.
current ratio and debt/equity ratio.
return on investment (ROI) and earnings per share (EPS).
debt/equity ratio and dividend payout ratio.

ROI and earnings per share would be directly affected by income taxes for the period. ROI is
calculated by taking net income and dividing it by average total assets. Net income is the
amount left over after taxes are paid. EPS is calculated by taking net income and subtracting
the preferred stock dividend, then dividing that total by the weighted average number of
common stock shares outstanding for the year.
Question 27:
2A2-LS50

All of the following are affected when merchandise is purchased on credit except:
*Source: Retired ICMA CMA Exam Questions.

current ratio.
total current assets.
net working capital.
total current liabilities.

When merchandise is purchased on credit, total current assets increase and total current
liabilities increase by the same amount . . . therefore, the net working capital (current assets −
current liabilities remains the same when merchandise is purchased on credit. The current
ratio, however, will change.
Question 28:
2A2-AT21

In the 20X3 fiscal year, Newman Manufacturing's gross profit margin remained unchanged
from the 20X2 fiscal year. But, in 20X3, the company's net profit margin declined from the
level reached in 20X2. This could have happened because in 20X3:
common share dividends increased.
cost of goods sold increased relative to sales.
corporate tax rates increased.
sales increased at a faster rate than operating expenses.

Increasing taxes will reduce net income after tax and thus the net profit margin.

Question 29:
2A2-CQ13

Lancaster Inc. had net accounts receivable of $168,000 and $147,000 at the beginning and
end of the year, respectively. The company's net income for the year was $204,000 on
$1,700,000 in total sales. Cash sales were 6% of total sales. Lancaster's average accounts
receivable turnover ratio for the year is:
10.87.
10.79.
10.15.
9.51.

The accounts receivable turnover per year is calculated as:
Accounts receivable turnover per year = (Net credit sales for year) / (average accounts
receivable balance for the year)
Average accounts receivable balance for the year = (beginning balance + ending balance) / 2
Average accounts receivable balance for the year = ($168,000 + $147,000) / 2 = $315,000 / 2
= $157,500
The net credit sales for the year is calculated by taking total sales of $1,700,000 and
multiplying it by one minus the percent of sales made in cash.
Net credit sales = $1,700,000(1 − 0.06) = $1,700,000(0.94) = $1,598,000
Accounts receivable turnover per year = $1,598,000 / $157,500 = 10.15 times.
Question 30:
2A2-AT24

A bondholder would be most concerned with which one of the following ratios?
Inventory turnover.
Times interest earned.
Earnings per share.
Quick ratio.

Times-interest-earned ratio reflects the company's ability to meet interest payments when they
are due. Since long-term creditors (bond holders) are interested in the company's long-run
solvency, the times-interest-earned ratio provides a valuable analysis.
Question 1:
2A2-CQ15

The following financial information is given for Anjuli Corporation (in millions of dollars).

Between the prior year and the current year, did the days sales in inventory and days sales in
receivables for Anjuli increase or decrease? Assume a 365-day year.
Days Sales in Inventory = Decreased ; Days Sales in Receivables = Increased
Days Sales in Inventory = Decreased ; Days Sales in Receivables = Decreased
Days Sales in Inventory = Increased ; Days Sales in Receivables = Decreased
Days Sales in Inventory = Increased ; Days Sales in Receivables = Increased

The days sales in inventory is calculated as:
Days sales in inventory = (# days in a year) / (inventory turnover)
Inventory turnover = (cost of goods sold (COGS) / (average inventory)
The COGS increased by 16.7%, which is calculated by taking the year-over-year change in
cost of goods sold, or (7 − 6) / 6 = 16.7%.
Inventory increased by 25%, which is also calculated by taking the year-over-year change in
inventory, or (5 − 4) / 4 = 25%.
Because both the COGS and the inventory increased, the inventory turnover would decrease,
resulting in an increase in the days sales in inventory.
Average number of days in accounts receivable = 365 / (the accounts receivable turnover per
year)
Accounts receivable turnover per year = (Net credit sales for year) / (average accounts
receivable balance for the year)
Since the sales increased by 10% ((11 − 10) / 10 = 10%), and the accounts receivable
increased by 33% ((4 − 3) / 4 = 33%), the accounts receivable turnover would decrease,
increasing the days sales in accounts receivable.
Question 2:
2A2-LS63

The dividend yield ratio is calculated by which one of the following methods?
*Source: Retired ICMA CMA Exam Questions.

Earnings per share divided by dividends per share.
Dividends per share divided by market price per share.
Market price per share divided by dividends per share.
Dividends per share divided by earnings per share.

The dividend yield ratio is calculated by taking dividends per share divided by market price per
share.
Question 3:
2A2-AT29

Birch Corporation had net income for the year of $101,504 and a simple capital structure
consisting of the following common shares outstanding.

Assume Birch Corporation issued a 20% stock dividend on August 1st. In this case, earnings
per share (EPS) (rounded to the nearest cent) were:
$2.87.
$2.72.
$2.41.
$2.67.

EPS = (net income − the preferred stock dividend) / (weighted average number of common
stock shares outstanding)
Birch has no preferred stock.
Therefore, EPS = (net income) / (weighted average number of common stock shares
outstanding).
The weighted average number of common stock shares outstanding (W) is calculated as
follows:
W = (24,000 shares for 2 months) + (29,400 shares for 4 months) + (36,000 shares for 5
months) + (35,040 shares for 1 month)
W = (2/12)(24,000) + (4/12)(29,400) + (5/12)(36,000) + (1/12)(35,040) = 4,000 + 9,800 +
15,000 + 2,940 = 31,720 shares.

A stock dividend or split occurring during the year is assumed to have occurred at the
beginning of the year when calculating W.
W, therefore, = 31,720 shares (1.20 stock dividend factor) = 38,064 shares.
EPS = ($101,504) / (38,064 shares) = $2.67 per share.
Question 4:
2A2-CQ34

For the most recent fiscal period, Oakland Inc. paid a regular quarterly dividend of $0.20 per
share and had earnings of $3.20 per share. The market price of Oakland stock at the end of
the period was $40.00 per share. Oakland's dividend yield was:
2%.
1%.
6.25%.
0.50%.

The dividend yield on common stock is calculated as:
Dividend yield on common stock = (annual dividend per common share) / (market price of
common stock)
Annual dividend = (4 quarters)($0.20) = $0.80
Dividend yield = $0.80 / $40 = 0.02, or 2%.
Question 5:
2A2-AT44

The Dawson Corporation projects the following for the year.

If Dawson Corporation's common stock is expected to trade at a price/earnings (P/E) ratio of
eight, the market price per share (to the nearest dollar) would be:
$56.
$125.
$68.

$72.

The P/E ratio for a stock is the ratio of its market price to the corporation's earning per share
(EPS). The P/E ratio is given here, but it is necessary to determine the stock price using the
other available information.
The stock price, then, is $56, which is the EPS of $7, multiplied by the P/E of 8.
The EPS of $7 is calculated by taking the $14,000,000 in earnings to common shareholders
and dividing it by 2,000,000 common shares outstanding.
The earnings to common shareholders (E to CS) is calculated as follows:
EBIT = earnings before interest and taxes
E to CS = [(EBIT − Interest)(1 − Tax Rate)] − (Preferred Stock Dividend)
E to CS = [($35,000,000 − $5,000,000)(1 − 0.4)] −($ 4,000,000)
= [$30,000,000(0.6)] −($ 4,000,000)
= $18,000,000 − $4,000,000 = $14,000,000.
Question 6:
2A2-CQ01

Broomall Corporation has decided to include certain financial ratios in its year-end annual
report to shareholders. Selected information relating to its most recent fiscal year is provided
below.

Broomall's working capital at year end is:
$37,000.
$10,000.
$28,000.

$40,000.

The term working capital as used by accountants is calculated by subtracting current liabilities
from current assets.
Working capital = current assets − current liabilities
Current assets include cash, accounts receivable, prepaid expenses, inventories, and
available-for-sale securities. (Available-for-sale securities are carried at fair value, not cost.)
Current assets = $10,000 + $20,000 + $8,000 + $30,000 + $12,000 = $80,000
Current liabilities include accounts payable and notes payable due in 90 days.
Current liabilities = $15,000 + $25,000 = $40,000
Working capital = $80,000 − $40,000 = $40,000.
Question 7:
2A2-CQ46

Morton Starley Investment Banking is working with the management of Kell Inc. in order to
take the company public in an initial public offering. Selected information for the year just
ended for Kell is as follows.

If public companies in Kell's industry are trading at a market to book ratio of 1.5, what is the
estimated value per share of Kell?
*Source: Retired ICMA CMA Exam Questions.

$27.50.
$21.50.
$13.50.

$16.50.

The market to book ratio is computed by taking the present market price of the stock, divided
by the book value of the firm's stock. The book value is computed by taking the total common
equity of $3,000,000, adding additional paid-in-capital of $24,000,000 and Retained Earnings
of $6,000,000, for a total of $33,000,000. Take the $33,000,000 and divide by the number of
shares outstanding $3,000,000 to get a book value of $11.00. Next, take the $11.00 book
value and multiply it by the ratio of 1.5 to get a estimated value per share of Kell of $16.50.
Question 8:
2A2-LS09

Capital structure is the:
mix of current and long-term assets, such as cash and fixed assets (plants and equipment).
terms a firm has on its equity, such as dividend payment schedules, stock repurchase agreements,
etc.
mix of equity, such as common stock, preferred stock, paid-in capital, and retained earnings.
mix of debt and equity the firm uses to finance operations and asset purchases.

Capital structure is the mix of long-term debt, on which interest and principal payments must
be made, and equity, in the form of common and preferred stock, which the firm uses to
finance operations.
Question 9:
2A2-CQ51

Roy company had 120,000 common shares and 100,000 preferred shares outstanding at the
close of the prior year. During the current year Roy repurchased 12,000 common shares on
March 1, sold 30,000 common shares on June 1, and sold an additional 60,000 common
shares on November 1. No change in preferred shares outstanding occurred during the year.
The number of shares of stock outstanding to be used in the calculation of basic earnings per
share at the end of the current year is:
*Source: Retired ICMA CMA Exam Questions.

100,000.
298,000.
137,500.
198,000.

The weighted average common shares calculation: (120,000 × 2/12) + (108,000 × 3/12) +
(138,000 × 5/12) + (198,000 x 2/12) = 20,000 + 27,000 + 57,500 + 33,000 = 137,500.
Question 10:
2A2-CQ49

Ray Company has 530,000 common shares outstanding at year-end. At December 31, for
basic earnings per share purposes, Ray computed its weighted average number of shares as

500,000. Prior to issuing its annual financial statements, but after year-end, Ray split its stock
2 for 1. Ray's weighted average number of shares to be used for computing annual basic
earnings per share is:
*Source: Retired ICMA CMA Exam Questions.

1,000,000.
1,060,000.
500,000.
530,000.

The weighted average number of shares would be doubled retroactively to adjust for the 2 for
1 stock split that occurred before the financial statements were issued.
Question 11:
2A2-AT47

Gartshore Inc. is a mail-order book company. The company recently changed its credit policy
in an attempt to increase sales. Gartshore's variable cost ratio is 70% and its required rate of
return is 12%. The company projects that annual sales will increase from the current level of
$360,000 to $432,000, but the average collection period on receivables will go from 30 to 40
days. Ignoring any tax implications, what is the cost of carrying the additional investment in
accounts receivable, using a 365-day year?
$2,160.
$1,492.
$1,761.
$2,601.

The cost of carrying the additional investment in accounts receivable is 12% of 70% of the
increase in the average accounts receivable balance. The increase in accounts receivable is
$17,761.
The new average accounts receivable balance is $47,342, which is calculated as:
$432,000 in sales divided by the accounts receivable turnover of 9.125 (365 days divided by
the collection period of 40 days).
The old average accounts receivable balance is $29,581, which is calculated as:
$360,000 in sales divided by the accounts receivable turnover of 12.17 (365 days divided by
the collection period of 30 days).
The difference between the old and new accounts receivable balances is an increase of
$17,761 (old = $29,581, new = $47,342), and this incremental amount is used to calculate the
increased carrying cost.

Increased carrying cost = (0.12)(0.7)($17,761) = $1,492.
Question 12:
2A2-AT30

Windsor Company had 720,000 shares of common stock outstanding on December 31, Year
1. An additional 240,000 shares of common stock were issued on April 1, Year 2, and 360,000
more on July 1, Year 2. On October 1, Year 2, Windsor issued 5,000, $1,000 face value, 7%
convertible bonds.
Each bond is convertible into 30 shares of common stock. The bonds were not considered
common stock equivalents at the time of their issuance, and no bonds were converted into
common stock in Year 2. If no other equity transactions will occur, the number of shares to be
used in computing primary earnings per share (EPS) and fully diluted earnings per share
(assuming dilution will occur), respectively, for the year ending December 31, Year 2, is:
1,080,000 and 1,117,500.
1,000,000 and 1,080,000.
1,117,500 and 1,117,500.
1,000,000 and 1,117,500.

The weighted average number of common stock shares outstanding (W) for primary (basic)
EPS is calculated as:
W = (20,000 shares for 3 months) + (720,000 + 240,000 shares for 3 months) + (960,000 +
360,000 for 6 months)
W = (3/12)(720,000) + (3/12)(960,000) + (6/12)(1,320,000)
W = 180,000 + 240,000 + 660,000
W = 1,080,000 shares
When calculating the weighted average number of common stock shares outstanding (W) for
fully diluted EPS, conversions of convertible securities and/or exercises of options or warrants
are assumed to have occurred at the earliest possible date.
For Windsor, the conversion of 5,000 bonds into 30 shares of common stock each (a total of
150,000 additional shares) is assumed to have occurred on October 1, Year 1. This results in
an additional 150,000 shares for 3 months or an additional weighted average number of
shares of (3/12)(150,000 shares) = 37,500 shares.
Therefore, the W = 1,080,000 shares + 37,500 shares = 1,117,500 shares.
1,080,000 shares will be used to calculate primary EPS and 1,117,500 shares will be used to
calculate fully diluted EPS.
Question 13:
2A2-LS51

Markowitz Company increased its allowance for uncollectable accounts. This adjustment will:
*Source: Retired ICMA CMA Exam Questions.

reduce the current ratio.
increase the acid test ratio.
increase working capital.
reduce debt-to-asset ratio.

The current ratio is calculated as current assets divided by current liabilities. By increasing the
allowance for doubtful accounts, the next effect is to reduce the current asset accounts
receivable. Therefore, this adjustment would reduce the current ratio.
Question 14:
2A2-CQ40

Globetrade is a retailer that buys virtually all of its merchandise from manufacturers in a
country experiencing significant inflation. Globetrade is considering changing its method of
inventory costing from first-in, first-out (FIFO) to last-in, first-out (LIFO). What effect would the
change from FIFO to LIFO have on Globetrade's current ratio and inventory turnover ratio?
*Source: Retired ICMA CMA Exam Questions.

Both the current ratio and the inventory turnover ratio would decrease.
The current ratio would decrease but the inventory turnover ratio would increase.
The current ratio would increase but the inventory turnover ratio would decrease.
Both the current ratio and the inventory turnover ratio would increase.

The value of inventory as a result of changing from FIFO to LIFO would decrease when prices
are rising. The effect of the change in inventory valuation would cause the current ratio to
decrease but the inventory turnover ratio would increase.
Question 15:
2A2-AT46

A change in credit policy has caused an increase in sales, an increase in discounts taken, a
decrease in the amount of bad debts, and a decrease in the investment in accounts
receivable. Based upon this information, the company's:
average collection period has decreased.
working capital has increased.
accounts receivable turnover has decreased.
percentage discount offered has decreased.

An organization's collection period is 365 days divided by its accounts receivable turnover.
The accounts receivable turnover is net sales divided by the average net accounts receivable

balance. Since the average receivable balance has decreased and sales have increased, the
accounts receivable turnover has increased. The increased turnover decreases the collection
period.
Question 16:
2A2-CQ50

On January 1, Esther Pharmaceuticals had a balance of 10,000 shares of common stock
outstanding. On June 1, the company issued an additional 2,000 shares of common stock for
cash. A total of 5,000 shares of 6%, $100 par, nonconvertible preferred stock was outstanding
all year. Esther's net income was $120,000 for the year. The earnings per share for the year
were:
*Source: Retired ICMA CMA Exam Questions.

$7.50.
$8.06.
$10.00.
$10.75.

Formula = (Net income − Preferred Dividends) / Weighted Average shares outstanding.
Weighted average shares outstanding (10,000 shares + (2,000 × 7 /12) = 11,167 weighted
average shares outstanding. (120,000 net income − 30,000 [5,000(100)(.060)] preferred
dividend) / 11,167 weighted average shares outstanding = 90,000/11,167 = $8.06 earnings
per share.
Question 17:
2A2-AT32

Devlin Company's acid-test ratio at May 31, Year 2, was:
2.04 to 1.
1.86 to 1.
0.60 to 1.
1.14 to 1.

The acid-test ratio (also known as the quick ratio) is the ratio of quick assets to current
liabilities. Quick assets are those easily converted to cash without significant loss. The quick
assets are cash, short-term investments (trading securities), and net receivables.
Devlin's acid-test ratio for Year 2 is calculated:
Acid-test ratio = (cash + marketable securities + net receivables) / current liabilities
Acid-test ratio = ($45 cash + $30 trading securities + $68 accounts receivable (net)) / ($125
current liabilities)
Acid-test ratio = $143 / $125 = ratio of 1.14 to 1.
Question 18:
2A2-LS16

If accounts payable is understated at the end of the year, which of the following statements is
correct?
The current ratio is understated.
The times interest earned ratio is understated.
The days' purchases in accounts payable is understated.
The long-term debt-to-equity ratio is understated.

If accounts payable is understated at the end of the year, then current liabilities are
understated. This would cause the current ratio to be overstated. The long-term debt-to-equity

ratio and the times interest earned ratio would not be affected by the problem. However, since
the days' purchases in accounts payable has average accounts payable in the numerator, this
ratio would be understated.
Question 19:
2A2-LS21

A company has $500,000 in current assets and $1,500,000 in fixed assets. Its paid-in capital
is $100,000 and retained earnings are $1,500,000. What is its debt ratio?
0.27.
0.15.
0.2.
5.

The debt ratio is calculated by dividing total debt by total assets. The company's total assets
are $2,000,000 ($1,500,000 fixed assets + $500,000 current assets). Total debt is assets less
equity or $400,000 ($2 million − $1.6 million). Therefore, its debt ratio is 0.2
($400,000/$2,000,000).
Question 20:
2A2-CQ47

Selected financial data for ABC Company is presented below.
ï‚· For the year just ended ABC has net income of $5,300,000.
ï‚· $5,500,000 of 7% convertible bonds were issued in the prior year at a face value of
$1,000. Each bond is convertible into 50 shares of common stock. No bonds were
converted during the current year.
ï‚· 50,000 shares of 10% cumulative preferred stock, par value $100, were issued in the
prior year. Preferred dividends were not declared in the current year, but were
current at the end of the prior year.
ï‚· At the beginning of the current year 1,060,000 shares of common stock were
outstanding.
On June 1 of the current year 60,000 shares of common stock were issued and sold.
ï‚· ABC's average income tax rate is 40%.
ABC Company's basic earnings per share for the current fiscal year is:
*Source: Retired ICMA CMA Exam Questions.

$4.29.
$4.38.
$3.67.

$4.73.

The shares outstanding is calculated as 1,095,000 = 1,060,000(5/12) + 1,120,000(7/12) =
441,667 + 653,333. The EPS formula = (Net income − Preferred Dividends) / Weighted
Average shares outstanding or (5,300,000 net income − 500,000 [50,000(100)(.1)] preferred
dividend)/ 1,095,000 weighted shares outstanding = 4,800,000/1,095,000 = $4.38 basic
earnings per share.
Question 21:
2A2-LS29

Assume that you examine the following selected ratios for JoJo. Inc.

Which of the following statements best describes an analysis of JoJo's ratios?
JoJo has excessive debt and manages that debt poorly; therefore, managers should attempt to
reduce debt usage.
JoJo has more debt than the industry average. While improvement is seen when compared to prior
years, JoJo should decrease debt usage further.
JoJo has improved in debt management; however, further reduction of debt is needed.
JoJo has more debt than the industry; however, the times interest earned ratio indicates that JoJo is
capable of managing its debt.

Note that JoJo's debt ratio is indicating less reliance on debt but that it has more debt than the
industry. The times interest earned ratio, however, is indicating that JoJo can pay its interest
payment 17 times over (compared to only 15 in industry). The combination of these two
factors indicates that JoJo is managing its debt well, even though the firm does have a little
more debt than industry average.
Question 22:
2A2-AT18

Which of the following is not a common problem associated with a relatively low inventory
turnover ratio?
High cost of funds may reduce profits.
Inventory may spoil or become obsolete.
Sales may be lost due to inventory outages.
Storage costs are high and reduce profits.

Inventory turnover is calculated by taking cost of goods sold (COGS) and dividing it by the
average inventory.
Inventory turnover = COGS / average inventory
A low turnover means that inventory is not selling quickly, and it also implies that there is a
large inventory. If the large inventory consists of the right items, buffer (safety) stocks will be
high, which would minimize inventory stock-outs or outages.
Question 23:
2A2-AT25

Last year, Johnson Company's days sales outstanding was 73 days. This year, days sales
outstanding is 91.25 days. Over the same time period, sales have declined by 20%. In this
period of time, what has happened to the level of Johnson Company's accounts receivable?
Accounts receivables have increased.
There is not enough information provided to make a determination.
Accounts receivable have decreased.
There has been no change in accounts receivable.

Let A/R = Accounts Receivable
Days sales outstanding = (A/R × 365) / Sales
Assume last year's sales = $100,000; then this year's sales = $80,000
73 = (A/R × 365) / $100,000; A/R = $20,000
91.25 = (A/R × 365) / $80,000; A/R = $20,000
The A/R amount is $20,000 in each scenario, therefore, there is no change in A/R.
Question 24:
2A2-LS47

A firm's total assets are $3,000,000 and its total equity is $2,000,000. If current assets
represent 50% of total assets and current liabilities represent 30% of total liabilities, then what
is the firm's current ratio?
3.
5.
10.
1.5.

The current ratio is current assets divided by current liabilities. $1,500,000/$300,000 = 5.

Question 25:
2A2-CQ12

Makay Corporation has decided to include certain financial ratios in its year-end annual report
to shareholders. Selected information relating to its most recent fiscal year is provided below.

Makay's average inventory turnover for the year was:
7.9 times.
5.4 times.
4.7 times.
5 times.

The inventory turnover per year is calculated as:
Inventory turnover per year = (cost of goods sold for year) / (average inventory balance for the
year)
Average inventory balance for the year = (beginning balance + ending balance) / 2
Average inventory balance for the year = ($26,000 + $30,000) / 2 = $56,000 / 2 = $28,000
Turnover per year = $140,000 / $28,000 = 5 times.
Question 26:
2A2-CQ17

A summary of the Income Statement of Sahara Company is shown below.

Based on the above information, Sahara's degree of financial leverage (DFL) is:

1.61.
0.96.
1.36.
2.27.

The DFL is calculated as:
DFL = EDIT/EBT
Operating income = sales − cost of sales − operating expenses
Operating income = $15,000,000 − $9,000,000 − $3,000,000 = $3,000,000
Earnings before tax (EBT) = operating income − interest expense
EBT = $3,000,000 − $800,000 = $2,200,000
DFL = $3,000,000/$2,200,000 = 1.36
(Note: Neither a preferred stock dividend nor tax rates are mentioned in the problem, so they
cannot be factored into the solution.)
Question 27:
2A2-LS28

SUL Company's cash balance increased by $400,000 during the current period. In examining
the statement of cash flow, you see that the net cash flow from investing was ($2.3 million)
and the net cash flow from financing was $2 million. Further investigation reveals that the firm
has fixed charges of $600,000, interest expenses of $525,000, and taxes of $120,000. What
is the operating cash flow to fixed charges ratio?
2.37.
1.24.
4.
2.5.

The Cash Flow to Fixed Charges Ratio = (Cash from Operations + Fixed Charges + Tax Payments) /
Fixed Charges and the Cash Flow from Operations can be derived from the net cash flow formula
where Net Cash Flow = Cash Flow from Operations + Cash Flow from Investing + Cash Flow from
Financing so $400,000 = CFO + (2.3M) + 2.0M and CFO = $700,000. Therefore the Cash Flow to
Fixed Charges Ratio = ($700,000 + $600,000 + $120,000) / $600,000 = 2.37.

Question 28:
2A2-AT42

Which one of the following factors would likely cause a firm to increase its use of debt
financing as measured by the debt-to-total-capitalization ratio?

an increase in the corporate income tax rate.
an increase in the degree of operating leverage.
increased economic uncertainty.
a decrease in times interest earned.

The tax deductibility of the interest on debt reduces the effective cost of the debt by (1 − tax
rate). The attractiveness of debt, therefore, increases with the tax rate.
Question 29:
2A2-AT02

Schodack Inc.'s common stock has a market price of $25, a price/earnings ratio of 7.2, and a
dividend yield of 5%. The earnings yield and dividend payout ratio, respectively, are closest to:
Earnings yield: 9%; Dividend payout ratio: 36%.
Earnings yield: 9%; Dividend payout ratio: 64%.
Earnings yield: 14%; Dividend payout ratio: 64%.
Earnings yield: 14%; Dividend payout ratio: 36%.

Question 30:
2A2-CQ33

Selected information regarding Dyle Corporation's outstanding equity is shown below.

Dyle's dividend yield on common stock is:
16.66%.
20.00%
11.11%.
16.88%.

The dividend yield on common stock is calculated as:
Dividend yield on common stock = (annual dividend per common share) / (market price of
common stock)
Dividend per common share = (total common stock dividend) / (number of common stock
shares outstanding)
Dividend per common share = ($700,000) / (350,000 shares) = $2
Dividend yield = $2 /$18 = 0.1111, or 11.11%.
Question 1:
2A2-AT01

The Dwyer Company balance sheet indicates that the company has $2,000,000 of 7.5%
convertible bonds, $1,000,000 of 9% preferred stock, par value $100, and $1,000,000
common stock, par value $10. The company reported net income of $317,000. The bonds can
be converted into 50,000 common shares. The income tax rate is 36%. Which one of the
following would Dwyer report as diluted earnings per share (EPS)?
$2.15.
$2.27.
$2.11.
$2.51.

EPS is calculated as:
EPS = (Net Income − Preferred Stock Dividend)/Weighted average # of common stock shares
Basic EPS ignores common stock equivalents (convertible securities, and stock options and
warrants). Fully-diluted EPS considers the dilution effect of the common stock equivalents.
To calculate fully-diluted EPS, adjust net income to reflect the conversion of the bonds and
adjust the weighted-average number of shares for the additional shares from the conversion.
The conversion of the bonds would add back to the net income the after-tax effect of the
interest.
The after-tax effect of the interest is one minus the tax rate, or 0.64, times the interest on the
bonds, or 7.5% of $2,000,000, which is $96,000.
After-tax effect of the interest = (1 − tax rate) × Bond interest = (1 − 36%) × ($2,000,000 ×
7.5%) = 0.64 × $150,000 = $96,000.
The conversion of the bonds results in 50,000 additional shares.
Fully-diluted EPS = (Net income − Preferred Stock Dividend + Tax effect of interest)/Weighted
average # of common stock shares + the additional shares from conversion
Fully-diluted EPS = ($317,000 − $90,000 + $96,000)/ (100,000 + 50,000)
= ($323,000)/(150,000) = $2.15
Fully-diluted EPS = $2.15.
Question 2:
2A2-CQ45

Bull & Bear Investment Banking is working with the management of Clark Inc. in order to take
the company public in an initial public offering. Selected financial information for Clark is as:

If public companies in Clark's industry are trading at twelve times earnings, what is the
estimated value per share of Clark?
*Source: Retired ICMA CMA Exam Questions.

$24.00.
$9.00.
$15.00.
$12.00.

First, we must calculate Earnings Per Share (EPS). To do so, we take the net income of
$3,750,000 and divide by the number of shares outstanding, 3,000,000 to get EPS of $1.25.
Take the EPS of $1.25 and multiply it by 12 to get $15.00.
Question 3:
2A2-AT26

CPZ Enterprises had the following account information.

The company has an operating cycle of five months.
The current ratio for CPZ Enterprises is:
2.14 to 1.
5.29 to 1.
5.00 to 1.
1.68 to 1.

The current ratio is calculated by taking current assets and dividing them by current liabilities.
Current assets consist of cash, short-term marketable securities, accounts receivable, shortterm notes receivable, inventories, and prepaid expenses. Current liabilities are those due
within one year, or one business cycle, whichever is longer. Current liabilities consist of
accounts payable, accrued expenses payable, interest expense payable, taxes payable,
short-term notes payable, and the current portion of long-term debt.

CPZ's current ratio is calculated as:
Current ratio = (current assets) / (current liabilities)
Current ratio = ($100,000 cash + $200,000 accounts receivable + $400,000 inventory +
$40,000 prepaid expenses) / ($80,000 accounts payable + $10,000 interest payable +
$50,000 notes payable (due in six months))
Current ratio = ($740,000) / ($140,000) = ratio of 5.29 to 1.
Question 4:
2A2-LS38

XYB Corporation has book value of $25 per share, net income of $3.3 million, annual
preferred dividends of $300,000, and 1 million common stock shares outstanding. The
company's common stock has a current market price of $45. What is the price to earnings
ratio for XYB Corporation's common stock?
15
1.8.
8.33.
0.067.

First, the calculation of earnings per share (EPS) is required. In this case, EPS is net income
available to the common stockholders divided by the number of shares of common stock
outstanding. Therefore, it is $3 per share (($3.3 million − $300,000)/1 million shares). The
price to earnings ratio is calculated by dividing the current market price by earnings per share.
$45 current market price ÷ $3 earnings per share = 15.
Question 5:
2A2-AT35

Assuming there are no preferred stock dividends in arrears, Devlin Company's return on
common shareholders' equity (ROE) for the year ended May 31, Year 2, was:
7.8%.
6.3%.
10.5%.
7.5%.

ROE is calculated as:
ROE = (net income − preferred stock dividends) / (average common shareholders' equity)
The average common shareholders' equity is derived by taking the beginning balance of
common shareholders' equity less the preferred stock dividend, and adding that to the ending
balance of common shareholders' equity less the preferred stock dividend, then dividing that
total by 2 to arrive at an average.
Devlin's average common shareholders' equity = [($585 +$574) / 2] − $150 = [($1,179)/2] −
$150 = $579.50 − $150 = $429.50
Devlin's preferred stock dividend is 6% of $150, or 0.06($150) = $9.
Net income is $54.
Therefore, ROE = ($54 − $9) / ($429.50) = $45 / $429.50 = 0.105, or 10.5%.
Question 6:
2A2-LS40

ABC Corporation has earnings per share of $5, pays a dividend of $1 per share, and has a
current market price of $60 per share. What is ABC Corporation's common stock dividend
yield?

1.7%.
8.3%.
1.2%.
20%.

The dividend yield is calculated by dividing the dividends per common share by the current
market price per share. $1 dividend per share ÷ $60 current market price = 0.017, or 1.7%.
Question 7:
2A2-CQ52

Selected information regarding Dyle Corporation's outstanding equity is shown below.

Dyle's yield on common stock is:
*Source: Retired ICMA CMA Exam Questions.

11.11%.
16.66%.
20%.
16.88%.

Dividend yield = Annual dividends per share/Market price per share. Annual dividends per
share is calculated by taking the common stock dividend paid of $700,000 and dividing it by
the 350,000 common shares outstanding, or $2.00. Dividing the $2.00 by the market price per
common share of $18, provides a dividend yield of 11.11%.
Question 8:
2A2-LS05

ZYX Company's annual statement reports sales of $16 million with a gross profit ratio of
37.5%. Beginning and ending inventory are $483,000 and $517,000, respectively. What is the
inventory turnover ratio?

10 times.
20 times.
2 times.
12 times.

Inventory turnover ratio is calculated by dividing cost of goods sold by the average inventory.
Recall that gross profit is Sales minus cost of goods sold (COGS). Given that the gross profit
ratio is 37.5%, then gross profit must be $6 million and COGS must be $10 million. Average
inventory is the average of beginning and ending inventory [(483,000 + 517,000)/2] =
$500,000. Therefore, the inventory ratio must be $10,000,000/$500,000 = 20 times. This
means that average inventory was sold 20 times during the year.
Question 9:
2A2-LS14

In which one of the following circumstances would operating leverage be likely to increase?
Firm purchases assets with debt.
Firm accepts a large order from a new customer. Excess capacity exists to fill the order.
Firm issues bonds to repurchase some of its own common stock.
Firm signs a contract to rent a new manufacturing site.

Operating leverage relates to the fixed operating costs of a firm. Of the choices given, the
rental of a new manufacturing site is the only one that will increase fixed operating costs.
Question 10:
2A2-CQ04

Davis Retail Inc. has total assets of $7,500,000 and a current ratio of 2.3 times before
purchasing $750,000 of merchandise on credit for resale. After this purchase, the current ratio
will:
be lower than 2.3 times.
remain at 2.3 times.
be higher than 2.3 times.
be exactly 2.53 times.

The current ratio is defined as:
Current ratio = (current assets) / (current liabilities)
Current assets include inventories, while current liabilities include accounts payable.
Assuming the current ratio of 2.3 is $2,300,000 in current assets divided by $1,000,000 in
current liabilities, the purchase of $750,000 in inventory on credit would change the ratio to
$3,050,000 / $1,750,000 = 1.74, which is lower than 2.3.

Question 11:
2A2-AT19

Given no other changes, which of the following actions would effectively increase a firm's
financial leverage index?
The firm issues new common stock and retires some of its long-term debt.
The firm purchases new long-term assets and pays for the purchase with long-term debt.
The firm reduces operating expenses by cutting employee benefits.
The firm refinances its existing long-term debt at a lower interest rate.

A measure of financial leverage is the ratio of total assets to common shareholders' equity.
Common shareholders' equity is calculated by taking total assets less total liabilities.
The purchase of assets by debt financing will increase total assets and total liabilities without
affecting common shareholders' equity. Therefore, the financial leverage ratio of total assets to
common shareholders' equity will increase.
Question 12:
2A2-LS08

Which of the following is not considered a weakness of ratio analysis?
A firm in multiple industries has difficulty comparing its ratios to any one industry.
Firms may use different valuation methods on inventory (FIFO, LIFO, Weighted-average, etc.).
Firms may use different depreciation methods (straight-line, double-declining balance, etc.).
Markets change from year to year, and comparing ratios over time is problematic.

While markets change from year to year, ratio analysis can still be valuable in analyzing how
the firm has changed with the markets. It is clear that conglomerates have difficulty comparing
ratios to a single industry and that accounting assumptions affect financial statements and
ratio analysis results.
Question 13:
2A2-AT08

Financial leverage results from the use of a source of funds for which the firm:
pays a variable return on each dollar amount raised.
earns a higher rate of return from its use than its cost.
pays a fixed percentage of revenue.
pays a fixed percentage of income.

Financial leverage results from financing asset acquisitions by borrowing at a rate that is less
than the rate that the firm can earn on its investments. It is sometimes referred to as, “trading
on the equity.”

Question 14:
2A2-LS24

ROK Company has $900,000 in current assets and $1,800,000 in fixed assets, including
$500,000 in goodwill and patents. Its paid-in capital is $100,000 and retained earnings are
$2,100,000. Eighty percent of liabilities are long-term liabilities. What is its net tangible assets
to long-term debt ratio?
0.23.
1.23.
3.6.
5.5.

The net tangible assets to long-term debt ratio is calculated by dividing net tangible assets by
long-term debt.
Net tangible assets for ROK Company are $2,200,000 ($900,000 current assets + $1,800,000
fixed assets − $500,000 intangible assets of goodwill and patents).
Total liabilities are assets less equity or $500,000 ($2.7 million − $2.2 million).
Since 80% of liabilities are long-term, total long-term liabilities are $400,000 (80% of
$500,000).
Therefore, the net tangible assets to long-term debt ratio is 5.5 ($2,200,000 net tangible
assets ÷ $400,000 long-term liabilities).
Question 15:
2A2-AT39

During the most recent fiscal year, Dongata Industries earned net income after tax of
$3,288,000. The company paid preferred share dividends of $488,000 and common share
dividends of $1,000,000. The current market price of Dongata's common shares is $56 per
share and the shares are trading at a price/earnings ratio of 8. How many common shares
does Dongata have outstanding?
400,000.
469,714.
411,000.
350,000.

Question 16:
2A2-AT33

Devlin Company's inventory turnover for the year ended May 31, Year 2, was:
3.67 times.
3.88 times.
5.33 times.
5.65 times.

Inventory turnover is calculated by dividing cost of goods sold (COGS) by the average
inventory. The average inventory, if not given, is calculated by adding together the beginning
and ending inventory balances and dividing the total by 2. Note that the beginning inventory
balance for Year 2 is the same as the ending inventory balance in Year 1.
Devlin's average inventory is calculated as:
Average inventory = = ($90 + $80) / (2) = $170/2 = $85
Devlin's inventory turnover for Year 2 is calculated as:
Inventory turnover = COGS / average inventory
Inventory turnover = $330/$85 = 3.88 times.
Question 17:
2A2-CQ20

The Liabilities and Shareholders' Equity section of Mica Corporation's Statement of Financial
Position is shown below.

Mica's debt/equity (D/E) ratio is:
33.9%.
25.1%.
32.2%.
25.6%.

The D/E ratio is the ratio of total liabilities to total stock equity at the end of the year.
In this scenario, total liabilities = accounts payable + accrued liabilities + 7% bonds payable
Total liabilities = $84,000 + $11,000 + $77,000 = $172,000
In this scenario, total stock equity = common stock + retained earnings + the reserve for bond
retirement
Total stock equity = $300,000 + $206,000 + $28,000 = $534,000
D/E ratio = $172,000 / $534,000 = 0.322 or 32.2%
The reserve is merely a reclassification of retained earnings.
Question 18:
2A2-LS33

Assume that a firm has a positive return on assets (ROA) that is less than 100%. What is the
effect on ROA if this firm purchases a new asset for cash and the result is an increase in net
income?
ROA is not calculated using net income.
ROA will decrease.
ROA will increase.
ROA will remain the same.

ROA is calculated by dividing net income by assets. If an asset is purchased for cash, the
transaction results in no change in assets. However, given that net income increases from this
asset, we have an increase in net income accompanied with no change in total assets.
Therefore, the ROA will increase.

Question 19:
2A2-LS44

At the end of the current year, a firm had a dividend yield of 3% and a price-to-earnings ratio
of 20X. If fully diluted earnings per share is $.72, what is the dividend paid on each share of
common stock?
$.22
$.43
$.60
$.14

Using the definitions of dividend yield (dividend paid per share of common stock divided by
the current price of the stock) and price to earnings ratio (current price of the stock divided by
fully diluted EPS), the product of the two ratios will yield dividend paid per share divided by
earnings per share. In this case, 3% x 20 = 0.6. Given that fully diluted EPS is $0.72, we can
solve for dividends per share (DPS) as 0.6 = DPS/0.72 and DPS = $0.43.
Question 20:
2A2-LS26

SUL Company's earnings for the year available for fixed charges are $1,500,000. Its fixed
charges are computed as $550,000 plus the annual preferred stock dividend. SUL has 10,000
shares of 10%, $50 par preferred stock outstanding. What is the earnings to fixed charges
ratio?
2.5.
2.73.
1.54.
6.67.

The earnings to fixed charges ratio is earnings available to meet fixed charges divided by
fixed charges. The analyst must determine which charges are included in fixed charges, as
items such as depreciation do not require cash payments. Interest expenses and preferred
stock dividends are part of fixed charges. Preferred stock dividends are computed as 10,000
shares × $5 (10% of par value) = $50,000. Therefore, total fixed charges are $550,000 +
$50,000 = $600,000. The earnings to fixed charges ratio for SUL Company is 2.5 ($1,500,000
earnings available to meet fixed charges ÷ $600,000 fixed charges).
Question 1:
2A2-CQ08

Lowell Corporation has decided to include certain financial ratios in its year-end annual report
to shareholders. Selected information relating to its most recent fiscal year is provided below.

Using a 365-day year, compute Lowell's accounts receivable turnover in days.
36.5 days.
39.8 days.
33.2 days.
26.1 days.

.
The accounts receivable turnover in days is calculated as:
Accounts receivable turnover in days = (365 days) / (the accounts receivable turnover per
year)
The accounts receivable turnover per year = (Net credit sales for year) / (average accounts
receivable balance for the year)
Average accounts receivable balance for the year = (beginning balance + ending balance)/ 2
Average accounts receivable balance for the year = ($20,000 + $24,000) / 2 = $44,000 / 2 =
$22,000
Turnover per year = $220,000 / $22,000 = 10 times.
Turnover in days = 365 days / 10 times per year = 36.5 days.
Question 2:
2A2-LS22

Which of the following ratios does not measure a firm's ability to use its assets efficiently?
Current ratio.
Days' sales in inventory.
Asset turnover rate.
Average collection period.

The current ratio measures liquidity while all the other ratios are measuring the firm's ability to
use its assets efficiently.
Question 3:
2A2-CQ07

Dedham Corporation has decided to include certain financial ratios in its year-end annual
report to shareholders. Selected information relating to its most recent fiscal year is provided
below.

Dedham's quick (acid-test) ratio at year end is:
2.00 to 1.
1.05 to 1.
1.80 to 1.
1.925 to 1.

The quick ratio, or acid test ratio, is defined as follows:
Quick ratio = (current assets − inventories − prepayments) / (current liabilities)
Current assets = (cash + receivables + pre-paid expenses + inventory + available-for-sale
securities at fair value)
Current Assets = $10,000 + $20,000 + $8,000 + $30,000 + 12,000 = $80,000
Current liabilities = (accounts payable + 90 day notes payable)
Current liabilities = $15,000 + $25,000 = $40,000
Quick Ratio = (Current assets − Inventory − Prepayments) / Current Liabilities
Quick Ratio = ($80,000 - $30,000 − $8,000) / $40,000
Quick Ratio = $42,000/$40,000 = 1.05 or 1.05 to 1.
Question 4:
2A2-LS61

Which one of the following is the best indicator of long-term debt paying ability?
*Source: Retired ICMA CMA Exam Questions.

Working capital turnover.
Current ratio.
Debt-to-total assets ratio.
Asset turnover.

The debt-to-total-assets ratio is an indicator of financial leverage. It tells you the percentage of
total assets that were financed by creditors, liabilities, and debt. This ratio is an indicator of the
long-term debt paying ability of a company. The other three measures are liquidity measures
dealing with short-term debt paying ability.
Question 5:
2A2-CQ21

Marble Savings Bank has received loan applications from three companies in the auto parts
manufacturing business and currently has the funds to grant only one of these requests.
Specific data, shown below, has been selected from these applications for review and
comparison with industry averages.

Based on the information above, select the strategy that should be the most beneficial to
Marble Savings.
Grant the loan to Bailey as all the company's data approximate the industry average.
Marble Savings Bank should not grant any loans as none of these companies represents a good
credit risk.
Grant the loan to Nutron as both the debt/equity (D/E) ratio and degree of financial leverage (DFL) are
below the industry average.
Grant the loan to Sonex as the company has the highest net profit margin and degree of financial
leverage.

The D/E ratio and the DFL both measure an organization's risk. The lower the debt/equity
ratio, the lower the risk. Similarly, the lower the degree of financial leverage, the lower the risk.

In the case of Nutron, both the D/E ratio and DFL measures are below the industry averages.
Therefore, it is the least risky of the three choices.
Question 6:
2A2-LS11

ABC Company has $1,000,000 in assets and owes $300,000. If ABC has sales of $10 million
and a net profit margin of 5%, what is its return on equity (ROE)?
7.1%.
50%.
71.4%.
70%.

ROE is net income divided by total equity. In this problem, equity (assets minus liabilities) is
$1 million − $300,000 = $700,000. If the profit margin (net income divided by sales) is 5% and
sales are $10 million, then net income must be $500,000. Therefore, ROE is
$500,000/$700,000 = 71.4%.
Question 7:
2A2-CQ10

Peggy Monahan, controller, has gathered the following information regarding Lampasso
Company.

Total sales for the year were $85,900, of which $62,400 were credit sales. The cost of goods
sold (COGS) was $24,500.
Lampasso's inventory turnover ratio for the year was:
8.2 times.
3.2 times.
8.9 times.
3.5 times.

The inventory turnover per year is calculated as:
Inventory turnover per year = (COGS for year) / (average inventory balance for the year)
Average inventory balance for the year = (beginning balance + ending balance) / 2
Average inventory balance for the year = ($6,400 + $7,600) / 2 = $14,000 / 2 = $7,000

Turnover per year = $24,500 / $7,000 = 3.5 times.
Question 8:
2A2-CQ27

On its year-end financial statements, Caper Corporation showed sales of $3,000,000, net
fixed assets of $1,300,000, and total assets of $2,000,000. The company's fixed asset
turnover is:
2.3 times.
1.5 times.
65.0%.
43.3%.

The fixed asset turnover is equal to net sales divided by the average net fixed assets for the
year.
Fixed asset turnover = (net sales) / (average net fixed assets for the year)
Fixed asset turnover = $3,000,000 / $1,300,000 = 2.3 times.
Question 9:
2A2-LS19

A company has $500,000 in current assets and $1,500,000 in fixed assets. Its paid-in capital
is $100,000 and retained earnings are $1,100,000. What is its debt-to-equity ratio?
0.27.
0.25.
0.67.
0.73.

The debt-to-equity ratio is calculated by dividing total debt by total equity. Total debt is
$800,000 (total assets of $2 million less total equity of $1.2 million) Therefore, the debt-toequity ratio is 0.67 ($800,000/$1,200,000).
Question 10:
2A2-AT04

The Statement of Financial Position for King Products Corporation for the fiscal years ended
June 30, Year 2, and June 30, Year 1, is presented below. Net sales and cost of goods sold
for the year ended June 30, Year 2, were $600,000 and $440,000, respectively.

King Products Corporation's inventory turnover for the fiscal year ended at June 30, Year 2,
was:
4.4.
6.
4.
5.

Inventory turnover is calculated by dividing cost of goods sold by the average inventory. The
average inventory, if not given, is calculated by adding together the beginning and ending
balances and dividing by 2. Note that the beginning inventory balance for year 2 is the same
as the ending inventory balance in year 1.
King's average inventory is calculated as:
Average inventory = ($120 + $100) / (2) = $220/2 = $110.
King's inventory turnover for year 2 is calculated as:
Inventory turnover = cost of goods sold / average inventory
Inventory turnover for year 2 = $440 / $110 = 4 times.
Question 11:
2A2-CQ23

Borglum Corporation is considering the acquisition of one of its parts suppliers and has been
reviewing the pertinent financial statements. Specific data, shown below, has been selected
from these statements for review and comparison with industry averages.

Borglum's objective for this acquisition is assuring a steady source of supply from a stable
company. Based on the information above, select the strategy that would fulfill Borglum's
objective.
Borglum should not acquire any of these firms as none of them represents a good risk.
Acquire Western as the company has the highest net profit margin and degree of financial leverage.
Acquire Rockland as both the debt/equity (D/E) ratio and degree of financial leverage (DFL) are below
the industry average.
Acquire Bond as both the debt/equity ratio and degree of financial leverage exceed the industry
average.

. The D/E ratio and the DFL leverage both measure an organization's risk. The lower the D/E
ratio, the lower the risk. Similarly, the lower the DFL, the lower the risk. In the case of
Rockland, both the D/E ratio and DFL least risky of the three choices.
Question 12:
2A2-AT03

Which one of the following transactions would increase the current ratio and decrease net
profit?
A federal income tax payment due from the previous year is paid.
Vacant land is sold for less than the net book value.
A long-term bond is retired before maturity at a discount.
Uncollectible accounts receivable are written off against the allowance account.

The current ratio is calculated by taking current assets and dividing them by current liabilities.
Net income is revenue and gains less expenses, losses, and taxes. A sale of land for less
than its net book value creates a loss, lowering net income. In addition, the sale increases
cash and, consequently, current assets. Since current liabilities are not affected, the current
ratio increases.
Question 13:
2A2-CQ37

The capital structure of four corporations is as follows.

Which corporation is the most highly leveraged?
Pane.
Warwick.
Cooper.
Sterling.

Leverage involves the use of non-common stock equity to enhance the return on common
stock equity. The most highly leveraged firm would be the one with the lowest percent of
common equity in its capital structure. Sterling has the lowest percent of common equity in its
capital structure.
Question 14:
2A2-LS48

A firm's total assets are $8,000,000, its total liabilities are $3,000,000, its current assets are
$1,700,000 ($400,000 cash, $200,000 money market fund, $700,000 inventory, $200,000
receivables, and $200,000 stocks), its current liabilities are $1,100,000 ($500,000 accounts
payable, $600,000 notes payable), and its equity is $5,000,000. What is the firm's cash to
current liabilities ratio?

1.545.
0.727.
0.545.
2.670.

The cash to current liabilities ratio is calculated by dividing the total of cash and cash
equivalents and marketable securities by current liabilities. ($400,000 cash + $200,000 money
market fund + $200,000 stocks) / $1,100,000 current liabilities = $800,000/$1,100,000 =
0.727.
Question 15:
2A2-AT20

Information concerning Hamilton's common stock is presented below for the fiscal year that
ended May 31.

The basic price/earnings (P/E) ratio for Hamilton's common stock is:
4 times.
5 times.
6 times.
3 times.

The P/E ratio for a common stock is the ratio of the common stock price to the latest primary
earnings per share (EPS).
P/E ratio = (market price per share) / (primary EPS)
Hamilton's P/E ratio = ($45.00) / ($11.25) = 4 times.
Question 16:
2A2-LS42

Which of the following statements is true?
Book value is usually higher than market value and is calculated by dividing net earnings by the
number of common shares outstanding.
Book value is usually lower than market value and is calculated by dividing net earnings by the

number of common shares outstanding.
Book value is usually lower than market value and is calculated by dividing the common shareholders'
equity by the number of common shares outstanding.
Book value is usually higher than market value and is calculated by dividing the common
shareholders' equity by the number of common shares outstanding.

Book value is usually lower than market value because investors trade on the potential
earnings of the company, not just its current book value. Book value is calculated by dividing
the common shareholders' equity by the number of common shares outstanding.
Question 17:
2A2-LS31

MON Corporation has total assets of $7,000,000. Its common equity is $3,000,000, including
paid-in capital and retained earnings. The company's net profit margin is 5% with revenues of
$13,050,000, and its preferred dividends are $25,000. What is MON Corporation's return on
common equity ratio?
0.31.
0.36.
0.28.
0.21.

The return on common equity ratio is calculated by dividing net income minus preferred
dividends by common equity. Net income is computed as 5% of $13,050,000 = $652,500.
Income available for common shareholders (net income minus preferred dividends) for MON
Corporation is $627,500 ($652,500 net income − $25,000 preferred dividends). Its return on
common equity ratio is 0.209 ($627,500 net income ÷ $3,000,000 common equity).
Question 18:
2A2-CQ18

A degree of operating leverage (DOL) of 3 at 5,000 units means that a:
1% change in earnings before interest and taxes will cause a 3% change in sales.
3% change in sales will cause a 3% change in earnings before interest and taxes.
1% change in sales will cause a 3% change in earnings before interest and taxes.
3% change in earnings before interest and taxes will cause a 3% change in sales.

The DOL is calculated as the percent change in operating income (also called earnings before
interest and taxes, or EBIT) divided by the percent change in sales.
DOL = (% change in EBIT) / (% change in sales)

DOL is given as 3. DOL is used to gauge the expected change in EBIT as a result of a change
in sales. Given a DOL of 3, a 1% change in sales will result in a 3% (DOL of 3 multiplied by
1% change in sales) change in EBIT.
Question 19:
2A2-AT10

If a company is profitable and is effectively using leverage, which one of the following ratios is
likely to be the largest?
return on total assets (ROA).
return on total shareholders' equity.
return on investment (ROI).
return on common equity (ROE).

ROE is equal to ROA multiplied by the leverage ratio. The leverage ratio is the ratio of total
assets to common stock equity. The leverage ratio is greater than one if leverage is effective.
ROA is also called ROI. Total shareholders' equity includes preferred stock, which is part of
the financial leverage.
Question 20:
2A2-LS27

SUL Company's net earnings for the year are $1,500,000. Its fixed charges are $645,000,
which includes interest expenses of $525,000 per year. Its taxes are $120,000. What is the
times interest earned ratio?
4.62.
2.5.
4.09.
2.87.

The times interest earned ratio is calculated by dividing earnings before interest and taxes
(EBIT) by the interest expense. In this example, EBIT needs to be calculated by adding the
interest and tax expenses to net earnings. EBIT is $2,145,000 ($1,500,000 net earnings +
$525,000 interest expense + $120,000 taxes). The times interest earned ratio is 4.0857
($2,145,000 EBIT ÷ $525,000 interest expenses).
Question 21:
2A2-LS46

A firm's total assets are $3,000,000 and its total equity is $2,000,000. If current assets
represent 50% of total assets and current liabilities represent 30% of total liabilities, then what
is the firm's working capital?
$500,000.
$4,000,000.

$1,800,000.
$1,200,000.

Working capital is current assets minus current liabilities: $1,500,000 − $300,000 =
$1,200,000.
Question 22:
2A2-LS06

LMN Company has beginning and ending accounts receivable of $238,000 and $262,000,
respectively. Its annual sales were $5 million, of which 62.5% were on credit. What is LMN
Company's days' credit sales in receivables?
29.2 days.
12.5 days.
9.2 days.
10 days.

Days' sales in receivables is calculated by dividing accounts receivable by annual credit sales
and multiplying that amount by 365. Average accounts receivable is the average of $238,000
and $262,000 or $250,000. Annual credit sales are 62.5% of $5,000,000 or $3,125,000.
Therefore, days' sales in receivables is ($250,000/$3,125,000) × 365 = 0.08 × 365 = 29.2
days.

Sponsor Documents

Or use your account on DocShare.tips

Hide

Forgot your password?

Or register your new account on DocShare.tips

Hide

Lost your password? Please enter your email address. You will receive a link to create a new password.

Back to log-in

Close